Descargar como pdf o txt
Descargar como pdf o txt
Está en la página 1de 72

ENTRENAMIENTO PARA LA ETAPA DE ZONA IIS AMIR MADRID

COMBINATORIA (PROBLEMAS INTRODUCTORIOS) 11/Noviembre/2014

La combinatoria es la rama de la matemática que estudia los diversos arreglos o selecciones


que podemos formar con los elementos de un conjunto dado, los cuales nos permite resolver
muchos problemas prácticos. Por ejemplo podemos averiguar cuántos números diferentes de
teléfonos, placas o loterías se pueden formar utilizando un conjunto dado de letras y dígitos.
Algunas de las temáticas a tratar serán:
• Hallar el número de formas diferentes en que podemos sacar algunos objetos después de
otros.
• Contar efectivamente el número de elementos de un conjunto que cumplan ciertas
condiciones.
• Hacer énfasis en conocer determinadas técnicas o estrategias de conteo que faciliten el
resolver problemas de permutaciones y combinaciones.

En la mayoría de los problemas de análisis combinatorio se observa que una operación o


actividad aparece en forma repetitiva y es necesario conocer las formas o maneras que se puede
realizar dicha operación.
Determinar el número de elementos que verifican una propiedad sin construirlos
todos. Contar sin enumerar
Para dichos casos es útil conocer determinadas técnicas o estrategias de conteo que facilitarán el
cálculo señalado. Ejemplos:
• Señalar las maneras diferentes de vestir de una persona, utilizando un número
determinado de prendas de vestir
• Ordenar 5 artículos en 7 casilleros.
• Contestar 7 preguntas de un examen de 10.
• Designar 5 personas de un total 50 para integrar una comisión.
• Sentarse en una fila de 5 asientos 4 personas.
• Escribir una palabra de 7 letras utilizando 4 consonantes y 3 vocales.

1. Cinco alumnos -Ángel, Beto, Carlos, Daniel y Enrique- van a participar en una
competencia, la cual consiste en realizar diferentes carreras uno contra uno. Cada uno de
los alumnos deberá correr contra todos los demás. ¿Cuál de los siguientes procedimientos
representa las carreras que se realizarán en la competencia?

2. ¿Cuántos cuadrados en total se encuentran en la siguiente


imagen?

3. ¿Cuántos números de 6 dígitos son tales que sus cifras


suman 21?

4. Una clase de tercero con 25 alumnos tiene que elegir


delegado y subdelegado. ¿Cuántas elecciones diferentes
son posibles? (Lección 01 Pág. 7)

5. Los 32 alumnos de una clase juegan un torneo de ajedrez. Di cuántas partidas se


celebrarán si: (Lección 01 Pág. 7)
a) Se juega en forma de liga.
b) Se juega por eliminatorias.
UNA COMIDA GRATIS

Diez jóvenes decidieron celebrar la terminación de sus estudios en la escuela secundaria con un almuerzo en
un restaurante. Una vez reunidos, se entabló entre ellos una discusión sobre el orden en que habían de
sentarse a la mesa. Unos propusieron que la colocación fuera por orden alfabético; otros, con arreglo a la
edad; otros, por los resultados de los exámenes; otros, por la estatura, etc. La discusión se prolongaba, la sopa
se enfrió y nadie se sentaba a la mesa. Los reconcilió el camarero, dirigiéndoles las siguientes palabras:

- Jóvenes amigos, dejen de discutir. Siéntense a la mesa en cualquier orden y escúchenme

Todos se sentaron sin seguir un orden determinado. El camarero continuó:

- Que uno cualquiera anote el orden en que están sentados ahora. Mañana vienen a comer y se sientan en
otro orden. Pasado mañana vienen de nuevo a comer y se sientan en orden distinto, y así sucesivamente hasta
que hayan probado todas las combinaciones posibles. Cuando llegue el día en que ustedes tengan que
sentarse de nuevo en la misma forma que ahora, les prometo solemnemente, que en lo sucesivo les convidaré
a comer gratis diariamente, sirviéndoles los platos más exquisitos y escogidos.

La proposición agradó a todos y fue aceptada. Acordaron reunirse cada día en aquel restaurante y probar
todos los modos distintos, posibles, de colocación alrededor de la mesa, con el objeto de disfrutar cuanto
antes de las comidas gratuitas.

Sin embargo, no lograron llegar hasta ese día. Y no porque el camarero no cumpliera su palabra sino porque el número
total de combinaciones diferentes alrededor de la mesa es extraordinariamente grande. Éstas son exactamente
3’628,800. Es fácil calcular, que este número de días son casi 10,000 años.

4.1. PRINCIPIOS BÁSICOS DE CONTEO


A menudo nos encontramos con preguntas del tipo ¿Qué proporción de...? ¿Cuál es la probabilidad de...? ¿De

cuántas maneras se puede...?

Muchas veces, para responder, se necesita un pensamiento sistemático y un poco de información adicional;
por ejemplo, ¿Cuántas rutas diferentes puedo usar para ir de Mérida a México? o ¿De cuántas maneras
pueden quedar los 3 primeros puestos en una carrera de 6 caballos?

Hay técnicas y principios matemáticos útiles en situaciones variadas, pero muchas preguntas se pueden
responder directamente, contando en forma sistemática, es decir, listando todos los posibles resultados en un
orden sistemático, para luego contar cuántos son, o desarrollando reglas de conteo. Algunas soluciones
parecen ingeniosas cuando se ven por primera vez (y muchas veces lo son) pero, como decía Juerguee Polya,
cuando podemos aplicar nuevamente estos métodos ingeniosos en problemas similares y en situaciones
relacionadas entre sí, hemos desarrollado una técnica.

Enunciaremos algunos principios que nos ayudarán a resolver muchísimos problemas de conteo, daremos
ejemplos de cómo usar estos principios y finalmente veremos algunos métodos menos rutinarios y más
ingeniosos.
ENTRENAMIENTO COMBINATORIA SECUNDARIA GUANAJUATO 2003

I. PRINCIPIO MULTIPLICATIVO

Si un proceso de selección se puede dividir en dos pasos consecutivos de modo que hay n
elecciones en el primero y por cada una de ellas hay m elecciones en el segundo, entonces el
número total de elecciones es n*m.
Ejemplo: En la etapa final de fútbol profesional de primera, cuatro equipos: CRISTAL (C), BOYS
(B), ESTUDIANTES (E), UNIVERSITARIO (U), disputan el primer y segundo lugar (campeón y
subcampeón). ¿De cuántas maneras diferentes estos equipos pueden ubicarse en dichos
lugares?
Solución: Utilizando el principio de multiplicación 4 x 3 = 12
EXPLICACIÓN:
1o El primer lugar puede ser ocupado por cualquiera de los cuatro equipos.
2o El segundo lugar puede ser ocupado por cualquiera de los otros tres equipos que restan
3o Por el principio de multiplicación, se observa que el evento del primer lugar se presenta de 4
maneras y el del segundo lugar de 3 maneras distintas, entonces el número de maneras totales
será: 4 x 3 = 12

1. En una fiesta hay pastel y gelatina. Hay 4 sabores de pastel y 9 de gelatina. Si te dicen:
“Elige un sabor de pastel y uno de gelatina para servir tu plato”, ¿de cuántas formas
puedes hacer tu elección? (Primaria)
2. El menú de un restaurante ofrece 3 platos calientes y 4 postres. ¿De cuántas maneras se
puede elegir un almuerzo de 1 plato caliente y 1 postre?
3. ¿Cuántas palabras de tres letras se pueden formar si se dispone de un alfabeto de dos
letras: a y b? (Nota: Son permisibles palabras como bba.)
4. ¿Cuántos códigos de una letra y un número de un dígito se pueden formar con las 26
letras del alfabeto y los números 0, 1, 2,...,9?
5. Hay cinco distintos tipos de tazas y tres de platos en una tienda. ¿De cuántas maneras se
puede comprar una taza y un plato?
6. En la tienda del problema anterior hay además 4 diferentes tipos de cucharas. ¿Cuántas
maneras hay de comprar una taza, un plato y una cuchara?
7. En el país de las maravillas hay tres pueblos: A, B y C. Existen seis caminos de A a B, y
cuatro de B a C. ¿De cuántas formas se puede ir desde A hasta C?
8. Un urbanista de una nueva subdivisión ofrece a los futuros compradores de una casa la
elección del estilo de la fachada entre Tudor, rústica, colonial y tradicional en una planta,
dos pisos y desniveles. ¿En cuántas formas diferentes un comprador puede ordenar una
de estas casas? Myers
9. Sam va a armar una computadora por sí mismo. Tiene la opción de comprar los chips
entre dos marcas, un disco duro de cuatro marcas, la memoria de tres marcas y un
conjunto de accesorios en cinco tiendas locales. ¿De cuántas formas diferentes puede
Sam comprar las partes? Myers
10.¿Cuántos números pares de cuatro dígitos se pueden formar con los dígitos 0, 1, 2 5, 6 y
9, si cada dígito se puede usar sólo una vez? Myers
Estos problemas ilustran lo que llamamos el principio multiplicativo: cuando una elección se
realiza consecuentemente, es decir tenemos que elegir una cosa y luego otra para completar (el
recorrido en el caso del viaje, por ejemplo), el total de formas es el producto de las posibilidades
para cada opción.

Si una operación se puede ejecutar en n1 formas, y si para cada una de éstas se puede llevar a
cabo una segunda operación en n2 formas, y para cada una de las primeras dos se puede
realizar una tercera operación en n3 formas, y así sucesivamente, entonces la serie de k
operaciones se puede realizar en n1n2 ・ ・ ・ nk formas.
II. PRINCIPIO ADITIVO
Si un proceso de selección se puede realizar de dos formas excluyentes de modo que la primera
de ellas admite n opciones y la segunda admite m opciones, entonces el número total de
selecciones posibles es n+m.
Ejemplo: Un repuesto de automóvil se venden en 6 tiendas en la Victoria o en 8 tiendas del
Madero. ¿De cuántas formas se puede adquirir el repuesto?
Solución: Por el principio de adición: Victoria o Madero 6 formas + 8 formas = 14 formas

1. En una fiesta hay pastel y gelatina. Hay 4 sabores de pastel y 9 de gelatina. Si te dicen:
“Elige un sabor de pastel o uno de gelatina para servir tu plato”, ¿de cuántas formas
puedes hacer tu elección?
2. Si Luci tiene 5 conjuntos deportivos y tiene 6 vestidos, ¿de cuántas maneras distintas se
puede vestir Luci?
3. Cinco empresas de transporte terrestre tienen servicio diario entre Mérida y México. Tres
empresas de aviación tienen vuelo diario entre Mérida y México. ¿Cuántas maneras hay de
ir de Mérida a México en avión o en autobús?
4. Si tengo una moneda de $50, una de $100, una de $200 y una moneda de $1000, ¿Cuál
es el número total de precios que puedo pagar usando alguna o todas mis monedas?
5. En el país de las maravillas se construyó un nuevo pueblo, llamado D, y se construyeron
también 3 caminos de A a D y 2 de D a C. ¿Cuántas formas hay ahora para ir de A a C?
6. ¿Cuántos grupos de 2 o más personas se pueden formar con 4 personas?
7. ¿Cuántos son los números enteros positivos de dos o tres dígitos?

Dividir el problema en casos es una idea muy útil. Por ejemplo, ayuda en la resolución del
siguiente problema:

8. Volvemos a la tienda que tiene cinco distintos tipos de tazas, tres de platos y cuatro de
cucharas. ¿De cuántas maneras se pueden acomodar dos cosas con nombres distintos?

Estos problemas ilustran lo que llamamos el principio aditivo: cuando una elección se puede
hacer de una manera o de otra (como en nuestro ejemplo, pasar por B o por D), el total de
formas es la suma de las posibilidades para cada opción.

9. Al cumpleañero de la fiesta anterior ya le compraste su regalo, pero falta la envoltura. La


caja en la que irá puede ser chica, mediana o grande. Si se elige la chica, ésta puede ser
de 5 colores diferente; si se elige la mediana, de 3 y si se elige la grande, de 6. Si
independientemente del tamaño, la caja puede llevar o no un moño, ¿de cuántas formas
diferentes puede ser la envoltura del regalo? (Primaria)

10.Si en el problema anterior el moño puede ser de 7 colores diferentes, ¿cuántas opciones
tenemos ahora para la envoltura del regalo? (Primaria)

El principio general es: “Si dos operaciones son mutuamente excluyentes (es decir, si solo una de ellas
puede ocurrir) y si la primera se puede hacer de n maneras diferentes y la segunda operación se puede
hacer de m maneras diferentes, entonces hay n + m maneras de realizar la primera o la segunda operación.”
Entrenamiento ONMAPS Guanajuato Primaria (Combinatoria)
11. Gaby tiene 15 blusas, 10 faldas, 9 pares de zapatos y 8 pares de botas. Cada artículo es
diferente de los demás. Se viste con una blusa, una falda y un par de alguno de los dos
tipos de calzado. ¿De cuántas formas se puede vestir?

12.Las ciudades de Nápoles, Venecia, Roma y Florencia están unidas entre ellas. A cada dos
de ellas las unen 7 caminos diferentes. ¿De cuántas formas se puede ir de Venecia a
Florencia sin pasar dos veces por la misma ciudad?

13.En un entrenamiento de tiro con arco, Susan debía incrustar una flecha en 8 frutas
diferentes. Si falló 3 de sus tiros, ¿de cuántas formas pudieron haber quedado las frutas?
(Una posible forma es: a las frutas 1, 4, 5, 7 y 8 les incrustó una flecha y a las frutas 2,3
y 6 les falló).

14.En el Bosque de los Mundos hay 5 estanques nuevos, 3 adultos y 8 viejos. Cada uno de
ellos conduce a un mundo distinto. Si Digory entra primero a un estanque nuevo, puede
regresar y luego entrar a otros 2: uno adulto y uno viejo, en cualquier orden. Si entra
primero a un estanque adulto, puede regresar y luego entrar a otro, mientras no sea
adulto, y si entra a uno viejo primero, ahí termina su recorrido. (Si por ejemplo entra
primero a un estanque adulto y luego decide entrar a uno nuevo, ya no puede elegir
visitar otros 2 mundos; las reglas descritas sólo aplican para el tipo de estanque que visita
primero). ¿De cuántas formas puede hacer Digory un recorrido?

15.Claudia quiere pintar las 5 paredes de su habitación. Ha comprado 12 colores diferentes


de pintura. ¿De cuántas formas puede pintar su habitación? ¿Y si no quiere que 2 paredes
juntas tengan el mismo color? ¿Y si no quiere 2 paredes del mismo color?

16.La calculadora de Ale perdió las teclas 0, 1 y 2. En ella, puede escribir un número de
hasta 8 dígitos. ¿Cuántos números distintos puede escribir en ella? ¿Y si quiere escribir un
número de 6 dígitos que tenga un único dígito 5? ¿Y si quiere uno de 4 dígitos que tenga
un único 5 y un único 4?

17.Chío quiere escribir un número de 9 dígitos sin usar 2 veces un mismo dígito. ¿Cuántos
números distintos puede escribir?

18.En la lotería se escoge una combinación de 6 números. Cada uno de ellos se elige de
entre el 1, 2, 3,…, 64, pudiéndose repetir. Trino cree que el dígito 3 es de mala suerte, así
que decide comprar un boleto en el que no aparezca. ¿Cuántas opciones de boletos tiene
Trino?

19.A, B, C y D toman cada uno una ficha de dominó. Notan que la cantidad de puntos en
cada ficha que tomaron es la misma. ¿De cuántas formas es esto posible?

A continuación presentamos una lista de problemas para resolver usando las ideas previas.

20.Problema 2.6 Decimos que un número es simpático si todos sus dígitos son impares.
¿Cuántos números simpáticos de seis cifras hay?

21.Problema 2.7 Arrojamos una moneda 3 veces. ¿Cuántas secuencias diferentes de “águila”
y “sello” podemos obtener?

22.Problema 2.8 Cada cuadro en una cuadrícula de 2×2 puede ser coloreado de blanco o de
negro. ¿Cuántas coloraciones diferentes de la cuadrícula existen?
23.Problema 2.9 ¿Cuántas maneras diferentes hay de llenar una planilla de pronósticos
deportivos? (En la planilla uno debe predecir los resultados de 13 juegos de futbol,
indicando ya sea la victoria para alguno de los dos equipos, o un empate).

24.Problema 2.10 El alfabeto hermitiano consiste únicamente de tres letras: A, B y C. Una


palabra en este lenguaje es una secuencia arbitraria de no más de cuatro letras. ¿Cuántas
palabras diferentes existen en este lenguaje?

Continuemos con otros problemas que nos ilustrarán otras estrategias distintas.

25.Problema 2.11 En un equipo de futbol con 11 integrantes se deben elegir un capitán


titular y un capitán suplente. ¿Cuántas maneras hay de hacer esto?

Este problema difiere de los anteriores en que la primera elección (la del capitán titular) afecta a
la segunda (la del capitán titular) porque una misma persona no puede ser ambas cosas a la
vez.

Seguimos con algunos problemas más en la misma dirección.

26.Problema 2.12 ¿Cuántas maneras distintas hay de fabricar una bandera tricolor con tres
tiras horizontales del mismo tamaño, si tenemos seis de esas tiras de colores distintos?
Podemos distinguir la parte de arriba de la bandera de la de abajo.

27.Problema 2.13 ¿Cuántas maneras distintas hay de colocar una torre blanca y una negra
en un tablero de ajedrez de modo que no se ataquen entre sí?

28.Problema 2.14 ¿Cuántas maneras distintas hay de colocar un rey blanco y uno negro en
un tablero de ajedrez de modo que no se ataquen entre sí?

29.Durante una campaña local, ocho candidatos republicanos y cinco demócratas se nominan
para presidentes del consejo electoral.
a) Si el presidente va a ser alguno de los candidatos, ¿Cuántos posibles presidentes hay?
b) ¿Cuántas posibilidades hay para que una pareja de candidatos (uno de cada partido) se
oponga entre sí en la elección final?

30.¿Cuántos partidos hay en un torneo de eliminación simple en el que participan n equipos


(para cada partido uno sale y el otro pasa a la siguiente etapa)?

31.¿Cuántas placas de automóvil distintas hay con 3 letras a la izquierda y 4 números a la


derecha? (Nota: Consideraremos el alfabeto de 27 letras castellanas.)

32.¿Cuántos números de cuatro cifras distintos impares hay? ¿Cuántos son pares?

33.Cinco estudiantes se escogen al azar de un grupo de 10 para formar una fila. ¿Cuántas
filas diferentes se pueden formar?

34.En una carrera compiten cinco corredores A; B; C; D; E. Si nunca hay empates, ¿En
cuántos resultados A le gana a B?

35.Seis personas A; B; C; D; E; F se sientan en torno a una mesa redonda. ¿Cuántas


posiciones circulares diferentes hay? (Dos posiciones se consideran iguales si una se
puede obtener de otra por rotaciones, Por ejemplo: ABCDEF es igual a FABCDE).
ENTRENAMIENTO COLIMA ONMAS 2008

36.¿De cuántas maneras se pueden formar en fila 5 estudiantes?

37.¿De cuántas maneras puede resultar un sorteo que consta de un primer premio y un
segundo premio en una clase de 25 alumnos?

38.¿Cuántos enteros entre 100 y 999 tienen todos sus dígitos distintos?

39.¿Cuántos números de 3 dígitos se pueden formar usando sólo los dígitos 3, 7 y 8? (Incluir
todos los números con dígitos repetidos).

SELECCIONES Con frecuencia cada uno de los pasos en que se divide un proceso de recuento
puede interpretarse como una elección o selección de k objetos elegidos entre los elementos
de un conjunto de n objetos.

Dado un conjunto de “n” elementos puede ocurrir:


1. Que los elementos sean distintos; en este caso, a los grupos se les denomina agrupaciones
simples.
2. Que algunos elementos sean iguales; en este caso, a los grupos se les denomina
agrupaciones con repetición.

Considerando la naturaleza de los elementos (que sean iguales o distintos), las agrupaciones
recibirán el nombre de permutaciones o combinaciones simples cuando no se repite ningún
elemento y permutaciones o combinaciones con repetición cuando algún elemento se repite.
Antes de continuar debemos explicar un concepto muy útil al trabajar con estas agrupaciones o
conjuntos: el concepto de factorial.

Definición de factorial. Para un entero n ≥ 1, n factorial, expresado n!, se define por:


n!= (n ) ⋅ (n − 1) ⋅ (n − 2) ⋅ ... ⋅ 3 ⋅ 2 ⋅ 1

El factorial para todo entero positivo n, el factorial de n o n factorial se define como el


producto de todos los números enteros positivos desde 1 (es decir, los números naturales) hasta
n. Por ejemplo, 5!=1x2x3x4x5=120

¿Y cuál es el factorial de cero? Una vez observado esto, podemos extender la definición de
factorial al 0, pues el número 0! tiene una interpretación natural: sólo hay una manera de
ordenar 0 objetos (que consiste en no hacer nada), y por lo tanto 0! = 1. El factorial de cero se
define así: 0! = 1

Gran parte de los problemas de combinatoria pueden plantearse como una serie de pasos cada
uno de los cuales consiste en elegir unos cuantos de entre ciertos elementos dados.

Es conveniente remarcar que, al hacer dicha selección, hay ocasiones en las que podremos
repetir dos veces el mismo objeto (por ejemplo, queremos escribir una palabra de 4 letras,
entonces debemos elegir cuatro de entre las 28 letras posibles, pero obviamente podemos
repetir dos veces la misma letra, como ocurre con la palabra "CASA") y otras ocasiones en las
que esto no será posible (si quiero elegir tres amigos para ir a cenar, no puedo escoger tres
veces al mismo). Así mismo y dependiendo de la situación, el orden en que escojo los elementos
a veces es importante y a veces no. Por ejemplo, si quiero escribir una palabra de 4 letras, el
orden de las mismas influye (no es lo mismo CASA que SACA), mientras que si quiero ir a cenar
con tres amigos, da igual el orden en que se los diga.

En general, siempre es más fácil resolver problemas en los que el orden es importante.
Veamos a continuación cómo se puede calcular el número de elecciones en cada caso.

PERMUTACIONES VS COMBINACIONES

Considérese un grupo de n individuos u objetos distintos (“distintos” significa que existe alguna
característica que diferencia a cualquier individuo u objeto de cualquier otro). ¿Cuántas
maneras existen de seleccionar un subconjunto de tamaño k del grupo? Por ejemplo, si
un equipo de ligas menores tiene 15 jugadores registrados, ¿cuántas maneras existen de seleccionar
9 jugadores para una alineación inicial? O si en su librero tiene 10 libros de misterio no
leídos y desea seleccionar 3 para llevarlos consigo en unas vacaciones cortas, ¿cuántas maneras
existen de hacerlo?
Una respuesta a la pregunta general que se acaba de plantear requiere distinguir entre
dos casos. En algunas situaciones, tal como el escenario del béisbol, el orden de la selección
es importante. Por ejemplo, con Ángela como lanzador y Ben como receptor se obtiene una
alineación diferente de aquella con Ángela como receptor y Ben como lanzador. A menudo,
sin embargo, el orden no es importante y a nadie le interesa qué individuos u objetos sean
seleccionados, como sería el caso en el escenario de selección de libros.

III. PERMUTACIONES

Con frecuencia nos interesamos en un espacio muestral que contiene como elementos a todas
las posibles ordenaciones o arreglos de un grupo de objetos. Por ejemplo, cuando queremos
saber cuántos arreglos diferentes son posibles para sentar a seis personas alrededor de una
mesa, o cuando nos preguntamos cuántas ordenaciones diferentes son posibles para sacar dos
billetes de lotería de un total de 20. Los diferentes arreglos se llaman permutaciones.

Se denomina permutación de n elementos a una selección ordenada de los n elementos


disponibles. La fórmula se obtiene usando el principio de multiplicación.
Es un caso particular de las variaciones
EJEMPLO: Determinar el número de formas distintas en que 5 personas pueden alinearse en la
taquilla de un cine

Calculemos ahora el número de maneras de ordenar n objetos en línea. Tales arreglos son
llamados permutaciones, y juegan un papel significativo en combinatoria y en álgebra.

CASO 1.- NO PODEMOS REPETIR (PERMUTACIÓN SIMPLE U ORDINARIA)


(PERMUTACIÓN SIMPLE DE N ELEMENTOS TOMADOS DE K EN K)
Se llama permutación simple de n elementos tomados de k en k (k < n) a los distintos grupos formados por k
elementos de forma que:
• Los k elementos que forman el grupo son distintos (no se repiten)
• Dos grupos son distintos si se diferencian en algún elemento o en el orden en que están colocados (influye el
orden).
• Aquí, no se utilizan todos los elementos.
Si elegimos un primer elemento, lo podemos hacer de n formas. Quitamos el elemento elegido y elegimos otro de
entre los n-1 que quedan. Esto podrá hacerse de n-1 formas. Quitamos también este elemento y nos quedamos con n-
2, de entre los que elegimos el tercero. Esto lo podremos hacer de n-2 formas...

Según la regla del producto, las maneras de escoger k elementos de entre un total de n según un determinado orden,
será igual al producto de: n ⋅ (n − 1) ⋅ (n − 2) ⋅ ... ⋅ (n − k + 1)

Notación. P denota el número de permutaciones de n elementos distintos tomados de k en k (tomados k a la vez).


n,k
Para llegar a una versión simplificada se opera así:

n (n − 1)(n − 2)(n − 3)...(n − (k − 1)) •


(n - k )(n - (k + 1)) ... (3)(2)(1) = n! = P
(n - k )(n - (k + 1)) ... (3)(2)(1) (n - k )! n ,k

En un año se otorgarán tres premios (a la investigación, la enseñanza y el servicio)


en un grupo de 25 estudiantes de posgrado del departamento de estadística. Si cada
estudiante puede recibir un premio como máximo, ¿cuántas selecciones posibles
habría? Myers
Como los premios son distinguibles, se trata de un problema de permutación. El
número total de puntos muestrales es

Supóngase, por ejemplo, que un colegio de ingeniería tiene siete departamentos,


denotados por a, b, c, d, e, f y g. Cada departamento tiene un representante en el consejo de
estudiantes del colegio. De estos siete representantes, uno tiene que ser elegido como presidente,
otro como vicepresidente y un tercero como secretario. ¿Cuántas maneras existen para seleccionar
los tres oficiales? Es decir, ¿cuántas permutaciones de tamaño 3 pueden ser formadas
con los 7 representantes? (Devore 62)

Existen diez asistentes de profesor disponibles para calificar exámenes en un curso de


cálculo en una gran universidad. El primer examen se compone de cuatro preguntas y el
profesor desea seleccionar un asistente diferente para calificar cada pregunta (sólo un asistente
por pregunta). ¿De cuántas maneras se pueden elegir los asistentes para calificar? (Devore 63)

Ejemplo 1. P10, 4 son las permutaciones de 10 elementos agrupándolos en subgrupos de 4 elementos:


10! 10 ⋅ 9 ⋅ 8 ⋅ 7 ⋅ 6 ⋅ 5 ⋅ 4 ⋅ 3 ⋅ 2 ⋅1
P10, 4 = = = 5,040
(10 − 4)! 6 ⋅ 5 ⋅ 4 ⋅ 3 ⋅ 2 ⋅1

Entonces podemos formar 5,040 subgrupos diferentes de 4 elementos a partir de los 10 elementos.

Ejemplo 2. ¿Cuántas banderas diferentes, de tres franjas horizontales de igual ancho y de colores distintos, pueden
confeccionarse a partir de siete colores diferentes?
Solución:
7!
P7,3 = = 210
4!

Ejemplo 3. ¿Cuántos números de tres cifras distintas se pueden formar con las nueve cifras significativas del sistema
decimal?
Al tratarse de números el orden importa y además nos dice "cifras distintas" luego no pueden repetirse:
P9,3 = 9 ⋅ 8 ⋅ 7 = 504

Por tanto, se pueden formar 504 números.

En el caso especial en que n = k, se llama permutaciones de n.


Se llaman permutaciones de n elementos a las diferentes agrupaciones de esos n elementos de forma que:
En cada grupo intervienen los n elementos sin repetirse ninguno (intervienen todos los elementos).
Dos grupos son diferentes si el orden de colocación de alguno de esos n elementos es distinto (influye el orden).
Notación: P denota el número de permutaciones de n elementos distintos.
n
n! n!
Pn = = = n!
(n − n )! 0!
Podemos probar que n objetos diferentes pueden ser ordenados uno detrás de otro de
n(n − 1)(n − 2)...(2)(1) formas, es decir, El número de permutaciones de n objetos es n!

Ejemplo 4. P10 son las permutaciones de 10 elementos:


¿De cuántas maneras se pueden sentar 10 comensales en una mesa?
P10 = 10!= 10 ⋅ 9 ⋅ 8 ⋅ 7 ⋅ 6 ⋅ 5 ⋅ 4 ⋅ 3 ⋅ 2 ⋅1 = 3'628,800

Ejemplo 5. Una madre tieene 3 hijos ¿de cuántas maneras distintas, nombrándolos uno por uno, puede llamarlos a
cenar? Solución: P3 = 3! = 6

Cualquier secuencia (finita) de letras del alfabeto español será llamada una palabra (sin importar
si es posible o no encontrarla en un diccionario). Por ejemplo, podemos formar seis palabras
usando las letras A, B y C cada una exactamente una vez: ABC, ACB, BAC, BCA, CAB y CBA.

Ejemplo 6. Calcular las maneras posibles de colocar las letras a, b, c.


P = 3! = 6 abc Acb

bac Bca

cab Cba

Ejemplo 7. Con las letras de la palabra DISCO ¿Cuántas palabras distintas se pueden formar?

Evidentemente, al tratarse de palabras el orden importa. Y además n = m, es decir tenemos que formar palabras de
cinco letras con cinco elementos D, I, S, C, O que no están repetidos.
P5 = 5!= 5 ⋅ 4 ⋅ 3 ⋅ 2 ⋅1 = 120
Por tanto, se pueden formar 120 palabras.

40.Problema 2.15 ¿Cuántos números de tres cifras se pueden formar usando los dígitos 1, 2
y 3 (sin repeticiones) en algún orden?

41.Problema 2.16 ¿Cuántas maneras hay de ordenar en una línea cuatro pelotas, de las
cuales una es roja, una negra, una azul y una verde?

PÁGINA 156 SERIE SHAUN

¿De cuántas maneras se pueden acomodar en línea 5 canicas de colores diferentes?


¿De cuántas maneras se pueden sentar 10 personas en una banca en la que sólo hay 4 asientos disponibles?
Se desea sentar en hilera a 5 hombres y 4 mujeres de manera que las mujeres ocupen los lugares pares. ¿De
cuántas maneras es posible hacer esto?
¿Cuántos números de cuatro dígitos se pueden formar con los 10 dígitos 0, 1, 2, 3, . . . , 9, si: a) puede haber
repeticiones, b) no puede haber repeticiones y c) no puede haber repeticiones y el último dígito debe ser
cero?
En un librero se van a acomodar cuatro libros diferentes de matemáticas, 6 libros diferentes de física y 2 libros
diferentes de química. ¿Cuántos son los acomodos posibles si: a) los libros de cada materia tienen que estar
juntos y b) sólo los libros de matemáticas tienen que estar juntos?
Cinco canicas rojas, 2 canicas blancas y 3 azules están ordenadas en línea. Si las canicas de un mismo color no
se distinguen unas de otras, ¿cuántas ordenaciones distintas se pueden tener?
¿De cuántas maneras pueden sentarse 7 personas a una mesa redonda si: a) las 7 se pueden sentar en cualquier
lugar y b) 2 determinadas personas no pueden sentarse juntas?

CASO 2) Permutación con repetición y NO existen elementos repetidos.


(Permutación con repetición de n elementos tomados de k en k).

Este caso es análogo al caso 1, sin más modificación que no quitar en cada paso los elementos
ya escogidos. Razonando igual se llega a que el número de posibles elecciones es:
n ⋅ n ⋅ n ⋅ ... ⋅ n = n k

Se llaman Permutaciones con repetición de n elementos tomados de k en k a los distintos grupos


formados por k elementos de manera que:

Los elementos que forman los grupos pueden estar repetidos.


Dos grupos son distintos si se diferencian en algún elemento o en el orden en que éstos están colocados (influye el
orden).

Notación. PR denota el número de permutaciones con repetición de n elementos distintos tomados de k en k.


n, k
PRn ,k = n k

Ejemplo 1. ¿Cuántos números de tres cifras pueden formarse con los dígitos 1 y 2
Solución: 23 = 8

Ejemplo 2: ¿Cuántos número de tres cifras se pueden formar con las nueve cifras significativas del sistema
decimal?
Al tratarse de números el orden importa y además no dice nada sobre “cifras distintas”, luego, sí pueden repetirse.

Por tanto, se pueden formar 729 números: PR9,3 = 9 3 = 729

Ejemplo 3. ¿Cuántas palabras distintas de 10 letras (con o sin sentido) se pueden escribir utilizando sólo las letras a,
b?
Al tratarse de palabras el orden importa y además como son palabras de 10 letras y sólo tenemos dos para formarlas,
deben repetirse.

PR10, 2 = 210 = 1024

Por tanto, se pueden formar 1024 palabras

Se van a elegir a un presidente y a un tesorero de un club estudiantil compuesto por


50 personas. ¿Cuántas opciones diferentes de funcionarios son posibles si (Myers 44)
a) no hay restricciones;
b) A participará sólo si él es el presidente;
c) B y C participarán juntos o no lo harán;
d) D y E no participarán juntos?

CASO 3) Permutación con repetición y existen elementos repetidos.

Son permutaciones con repetición de n elementos, no todos distintos. Todas las agrupaciones de
n elementos, formadas por aquellos, están dispuestas linealmente y sin que ninguno haga falta

Se denomina permutación con repetición de n elementos entre los que hay k grupos de
elementos indistinguibles de tamaños n1, …,nk, a una selección ordenada de los n elementos. Se
representa P(n, n1, …, nk) y se verifica que el número de permutaciones con repetición es:

Clave: para cada grupo de elementos indistinguibles solo importan los lugares que ocupan en
una alineación de los n elementos

Durante un entrenamiento del equipo de fútbol americano de la universidad, el


coordinador defensivo necesita tener a 10 jugadores parados en una fi la. Entre estos
10 jugadores, hay 1 de primer año, 2 de segundo año, 4 de tercer año y 3 de cuarto
año, respectivamente. ¿De cuántas formas diferentes se pueden arreglar en una fi la,
si sólo se distingue su nivel de clase? (Myers 45)

¿En cuántas formas se pueden asignar siete estudiantes de posgrado a una habitación
de hotel triple y a dos dobles, durante su asistencia a una conferencia? (Myers 46)

¿Cuántos arreglos diferentes de letras se pueden hacer con las letras de la palabra
STATISTICS? (Myers 47)
Ejemplo 1. Calcular las permutaciones de 10 elementos, en los que uno de ellos se repite en 2 ocasiones y otro se
repite en 3 ocasiones:

10!
Solución: P210,3 = = 302,400
2!×3!

Es decir, tendríamos 302,400 formas diferentes de agrupar estos 10 elementos.

Ejemplo 2. ¿Cuántos números de 6 cifras se pueden formar con los dígitos 1, 1, 1, 2, 2, 3?

Solución:
6!
= 60
3!×2!

Ejemplo 3. ¿De cuántas maneras distintas pueden colocarse en línea nueve bolas de las que 4 son blancas, 3
amarillas y 2 azules?

El orden importa por ser de distinto color, pero hay bolas del mismo color (están repetidas) y además n = k, es decir
colocamos 9 bolas en línea y tenemos 9 bolas para colocar:

Por tanto, tenemos 1260 modos de colocarlas.

En los siguientes cinco problemas se debe calcular el número de diferentes palabras (en el
sentido acordado) que se pueden obtener reacomodando las letras de una palabra en particular.

42.Problema 2.17 “VECTOR”

43.Problema 2.18 “MONOS”

44.Problema 2.19 “CAMADA”

45.Problema 2.20 “CERRADURA”

46.Problema 2.21 “CARACTERIZACIÓN”


Estos problemas acerca de palabras muestran una idea muy importante e interesante: la del
conteo múltiple. Esto es, en vez de contar el número de objetos en los que estamos interesados,
puede ser más fácil contar otros objetos cuya cantidad es un múltiplo conocido del número de
objetos que estamos buscando (generalmente, se trata, como en los ejemplos anteriores, de
contar cada uno de los objetos que buscamos varias veces, y luego dividir entre el número de
veces que contamos cada objeto).

He aquí más problemas para resolver usando este método.

47.Problema 2.22 Hay 20 pueblos en un cierto país, y cada par de ellos está conectado por
una carretera directa (es decir, que no pasa por ningún otro pueblo). ¿Cuántas carreteras
hay?

48.Problema 2.23 ¿Cuántas diagonales hay en un n-ágono? (Entendemos por diagonal


cualquier segmento que una dos vértices no consecutivos).

49.Problema 2.24 Entenderemos por “collar” una cadena circular con varias cuentas en ella.
Se permite rotarlo pero no voltearlo (es decir, si lo tenemos sobre una mesa, que la parte
que toca la mesa se vuelva la de arriba). ¿Cuántos collares diferentes se pueden hacer
usando 13 cuentas distintas?

50.Problema 2.25 Repetir el problema anterior, pero ahora con la condición de que sí
podemos voltearlo.

El siguiente problema ilustra otra importante idea combinatoria.

51.Problema 2.26 ¿Cuántos números de seis dígitos tienen al menos un dígito par?

La idea principal en esta solución fue usar el método del complemento, es decir, contar (o
considerar) los objetos “no pedidos” en vez de los “pedidos”, lo cual es más fácil en ciertos
casos. He aquí otro problema que puede ser resuelto usando este método.

52.Problema 2.27 Hay seis letras en el lenguaje hermitiano. Una palabra es cualquier
secuencia de seis letras entre las cuales hay (al menos) dos iguales. ¿Cuántas palabras
distintas hay en este lenguaje?

A continuación se presenta una lista de problemas diversos para ser resueltos utilizando las
estrategias aprendidas.

PROBLEMAS DE PRÁCTICA 1

53.Problema 2.28 En una oficina postal hay cinco tipos de sobres y 4 tipos de estampillas.
¿De cuántas formas puedes comprar un sobre y una estampilla?

54.Problema 2.29 ¿De cuántas formas puedes elegir una vocal y una consonante de la
palabra “olimpiada”?

55.Problema 2.30 Siete sustantivos, cinco verbos y dos adjetivos son escritos en el pizarrón.
Podemos formar una oración eligiendo una palabra de cada tipo y no nos importa el
sentido que tenga la oración. ¿De cuántas formas podemos hacer esto?

56.Problema 2.31 Cada uno de dos coleccionistas tiene 20 estampas y 10 postales. Decimos
que un intercambio es justo si ellos intercambian una estampa por una estampa o una
postal por una postal. ¿De cuántas formas podemos hacer un intercambio justo entre los
dos coleccionistas?
57.Problema 2.32 ¿Cuántos números de seis dígitos tienen todos sus dígitos de la misma
paridad (todos impares o todos pares)?

58.Problema 2.33 ¿De cuántas formas podemos enviar seis cartas urgentes si tenemos tres
mensajeros y cada carta puede dársele a cualquiera de los mensajeros?

59.Problema 2.34 ¿De cuántas maneras podemos elegir cuatro cartas de diferentes palos de
una baraja común de 52 cartas?

60.Problema 2.35 ¿De cuántas formas podemos acomodar todos o algunos de cinco libros en
un librero?

61.Problema 2.36 ¿De cuántas formas podemos acomodar ocho torres en un tablero de
ajedrez de tal forma que ninguna ataque a las otras?

62.Problema 2.37 Hay n hombres y n mujeres en una clase de baile. ¿De cuántas formas
podemos acomodarlos por parejas (un hombre con una mujer) para la clase?

63.Problema 2.38 Las reglas de un torneo de ajedrez dicen que cada concursante debe jugar
con todo otro concursante exactamente una vez. ¿Cuántos juegos se jugarán si hay 18
jugadores?

64.Problema 2.39 ¿De cuántas formas puedes poner:


a) dos alfiles
b) dos caballos
c) dos reinas
en un tablero de ajedrez de tal forma que una pieza no ataque a la otra?

65.Problema 2.40 Hay tres cuartos en un dormitorio: uno sencillo, uno doble y uno
cuádruple. ¿De cuántas formas se pueden acomodar siete estudiantes en el dormitorio?

66.Problema 2.41 Una madre tiene dos manzanas, tres peras y cuatro naranjas. Cada
mañana por nueve días ella da una fruta a su hijo. ¿De cuántas formas puede hacer esto?

67.Problema 2.42 ¿De cuántas formas se puede poner el conjunto de las piezas de ajedrez en
la primera fila del tablero? (recuerda que son un rey, una reina, dos torres idénticas, dos
caballos idénticos y dos alfiles idénticos).

68.Problema 2.43 ¿Cuántas palabras se pueden crear usando exactamente cinco letras A y
no más de tres letras B?

69.Problema 2.44 ¿Cuántos números de diez dígitos tienen al menos dos dígitos iguales?

70.Problema 2.45 ¿Será cierto que los números de siete dígitos sin dígitos 1 en su
representación decimal constituyen más del 50% de todos los números de siete dígitos?

71.Problema 2.46 Se arroja un dado tres veces. De entre todos los posibles resultados,
¿cuántos tienen al menos una ocurrencia del seis?

72.Problema 2.47 ¿De cuántas formas se pueden dividir 14 personas en 7 parejas?


73.Problema 2.48 ¿Cuántos números de nueve dígitos tienen suma par de sus dígitos?

IV. COMBINACIONES

Un niño le pide a su mamá que le lleve cinco cartuchos de Game-BoyTM de su colección


de 10 juegos de arcada y 5 de deportes. ¿Cuántas maneras hay en que su mamá
le llevará 3 juegos de arcada y 2 de deportes, respectivamente? (Myers 46)

Una mano de bridge se compone de 13 cartas seleccionadas de entre un mazo de 52 cartas


sin importar el orden. ¿Cuántas manos de bridge diferentes existen? (Devore 64)

En muchos problemas nos interesamos en el número de formas de seleccionar r


objetos de n sin importar el orden. Tales selecciones se llaman combinaciones.

Una combinación es todo arreglo en el que no importa el orden o la posición de los elementos
que lo componen.

CASO 1.- EL ORDEN NO IMPORTA PERO NO SE PUEDEN REPETIR ELEMENTOS.

PÁGINA 158 SERIE SHAUN

¿De cuántas maneras pueden colocarse 10 objetos en dos grupos, uno de 4 y otro de 6 objetos?
¿De cuántas maneras puede formarse de un grupo de 9 personas un comité de 5 personas?
Con 5 matemáticos y 7 físicos hay que formar un comité que conste de 2 matemáticos y 3 físicos. ¿De cuántas
maneras se puede formar este comité si: a) puede incluirse a cualquiera de los matemáticos y a cualquiera de
los físicos, b) hay uno de los físicos que tiene que formar parte del comité y c) hay dos de los matemáticos que
no pueden formar parte del comité?
Una niña tiene 5 flores que son todas distintas. ¿Cuántos ramos puede formar?
Con 7 consonantes y 5 vocales ¿cuántas palabras con 4 consonantes distintas y 3 vocales distintas pueden
formarse? No importa que las palabras no tengan significado.
Combinación de r elementos
Se denomina combinación de r elementos de un total de n a una selección, sin orden, de r
elementos distintos de los n posibles. Se representa C(n,r) y se verifica que el número de
combinaciones de n elementos tomados de r en r es

(“De n se eligen r”)

EJEMPLO: Determinar los colores que se pueden obtener al mezclar 3 colores distintos de una
paleta de 8 colores
A) Combinación con repetición de r elementos
Se denomina combinación con repetición de r elementos de un total de n a una selección, sin
orden, de r elementos posiblemente repetidos de los n posibles.
Se representa CR(n,r) y se verifica que el número de combinaciones con repetición de n
elementos tomados de r en r es:

EJEMPLO: Determinar el número de pedidos diferentes que puede recibir un camarero que
atiende a una mesa de 5 personas si las bebidas disponibles son café, té o menta.
La diferencia básica entre permutación y combinación es que para la permutación el orden si
importa es decir, que cuenta todos los órdenes distintos de una misma lista; para la
combinación, por otro lado, el orden no importa, es decir únicamente importa la lista y no sus
órdenes distintos.

Ejemplo 1. Un alumno decide rendir tres de los cinco exámenes finales ¿De cuántas maneras distintas puede elegir
esas tres pruebas?

Solución:
5!
C 5,3 = = 10
3!×2!

Ejemplo 2. ¿Cuántas combinaciones de 6 aciertos existen en la lotería primitiva?

 49  49!
C 49,6 =   = = 13,983,816
 6  6!(49 − 6)!

Es decir, que tendríamos que echar 13’983,816 apuestas de 6 números para tener la seguridad al 100% de que
íbamos a acertar.

Ejemplo 3. ¿Cuántos grupos de 5 alumnos pueden formarse con los treinta alumnos de una clase? (Un grupo es
distinto de otro si se diferencia de otro por lo menos en un alumno)
No importa el orden (son grupos de alumnos). No puede haber dos alumnos iguales en un grupo evidentemente,
luego sin repetición.

 30  30! 30 ⋅ 29 ⋅ 28 ⋅ 27 ⋅ 26 ⋅ 25!
C 30,5 =   = = = 142,506
 5  5!(30 − 5)! 5!⋅25!

Por tanto, se pueden formar 142,506 grupos distintos.

n
En general, calcular   por la fórmula anterior implica calcular varios factoriales, lo que hace que no sea
k 
muy útil en la práctica. Un método alternativo viene dado por las siguientes propiedades:

Proposición.

n n
1)   =   = 1
0 n

 n   n − 1   n − 1
2)   =   +  
 k   k − 1  k 

CASO 2.- EL ORDEN NO IMPORTA Y SÍ SE PUEDE REPETIR (COMBINACIONES CON REPETICIÓN).


Una combinación con repetición de tamaño k es una selección no ordenada de k objetos elegidos entre n
tipos diferentes de objetos, habiendo una cantidad ilimitada de cada tipo.

Una combinación con repetición puede describirse diciendo que elegimos x1 objetos de tipo 1, x2 objetos de
tipo 2,..., xn objetos de tipo n para alguna n-pla (x1, x2,..., xn). Cada uno de los enteros x1, x2,..., xn es no
negativo y x1 + x 2 + ... + x n = k . Así pues, las combinaciones con repetición de tamaño k se corresponden con
las soluciones enteras no negativas de la ecuación: x1 + x 2 + ... + x n = k

El número de combinaciones de tamaño k con repetición ilimitada elegidas entre n tipos diferentes de objetos
es:

n −1+ k 
C nR,k =  
 k 
Cada combinación con repetición se representa por una palabra en el alfabeto {0,1} del siguiente modo: Los
0’s son las marcas que separan los objetos de cada tipo y los 1’s indican los objetos que hay de cada uno de los
tipos entre dos marcas consecutivas. Si hay n tipos de objetos se necesitan n - 1 marcas para separar los tipos
y, por tanto, las palabras de 0’s y 1’s tienen longitud n - 1 + k. Así se convierte cada combinación con
repetición de tamaño k en una combinación de k objetos (las posiciones de los 1’s) elegidos entre un conjunto
de n - 1 + k elementos (las posiciones).

Se llama combinaciones con repetición de n elementos tomados de k en k, a los distintos grupos formados por
k elementos de manera que:

• Los elementos que forman cada grupo pueden estar repetidos.


• Dos agrupaciones distintas se diferencian al menos en un elemento, sin tener en cuenta el orden.
Ejemplo 1. C10R , 4 son las combinaciones de 10 elementos con repetición, agrupándolos en subgrupos de 4, en
los que 2, 3 o los 4 elementos podrían estar repetidos:

13! 13 ⋅12 ⋅11⋅10 ⋅ 9 ⋅ 8 ⋅ 7 ⋅ 6 ⋅ 5 ⋅ 4 ⋅ 3 ⋅ 2 ⋅1


C10R , 4 = = = 715
4!×9! (4 ⋅ 3 ⋅ 2 ⋅1) ⋅ (9 ⋅ 8 ⋅ 7 ⋅ 6 ⋅ 5 ⋅ 4 ⋅ 3 ⋅ 2 ⋅1)

Es decir, podríamos formar 715 subgrupos diferentes de 4 elementos.


Ejemplo 2. Las combinaciones con repetición de los elementos {a, b, c, d} tomados de dos en dos son: aa, ab,
ac, ad, bb, bc, bd, cc, cd, dd

Ejemplo 3. En una bodega hay 12 botellas de ron, 12 de ginebra y 12 de anís. Un cliente compró 8 botellas en
total. ¿Cuántas posibilidades hay?

C3R,8 = 120

Ejemplo 4: En una confitería hay cinco tipos diferentes de pasteles. ¿De cuántas formas se pueden elegir
cuatro pasteles?

No importa el orden (son pasteles). Puede haber dos o más pasteles del mismo tipo en un grupo, luego con
repetición.

 5 + 4 − 1 8! 8 ⋅ 7 ⋅ 6 ⋅ 5 ⋅ 4!
C 5R, 4 =   = = = 70
 4  4!(5 − 1)! 4!⋅4!

Por tanto, se pueden elegir 4 pasteles de 70 formas distintas.

SELECCIONES (de k elementos entre n)

ORDENADAS NO ORDENADAS
n
SIN REPETICIÓN n ⋅ (n − 1) ⋅ (n − 2 ) ⋅ ... ⋅ (n − k + 1)  
k 

nk n −1+ k 
CON REPETICIÓN  
 k 

Problema 2.49 Se deben elegir dos estudiantes de entre un grupo de treinta para un concurso de
matem´aticas. ¿De cu´antas maneras se puede hacer esto?

Problema 2.50 ¿Cu´antas maneras hay de elegir un equipo de 3 estudiantes de entre un total de
30?

Problema 2.51 ¿De cu´antas maneras se pueden elegir cuatro colores de entre 7 colores dados?

Problema 2.52 Un estudiante tiene 6 libros de matem´aticas, y otro tiene 8. ¿Cu´antas maneras
hay de intercambiar 3 libros del primer estudiante por 3 del otro?

Problema 2.53 Hay dos mujeres y 7 hombres en un club de ajedrez. Debe elegirse un equipo de
4 personas para un torneo, el cual debe incluir por lo menos a una mujer. ¿De cu´antas
maneras se
puede hacer esto?

Problema 2.54 ¿Cu´antas maneras hay de dividir a un grupo de 10 personas en dos equipos de
basquetbol de 5 personas cada uno?

Problema 2.55 Diez puntos est´an marcados en el plano de tal manera que no hay tres
colineales
(es decir, no hay tres de ellos que est´en sobre la misma l´ınea recta). ¿Cu´antos tri´angulos
hay con
v´ertices en estos puntos?

Problema 2.56 Un escuadr´on especial consiste de 3 oficiales, 6 sargentos y 60 soldados rasos.


¿De
cu´antas maneras se puede elegir a un grupo de 1 oficial, 2 sargentos y 20 soldados rasos para
una
misi´on?

Problema 2.57 Diez puntos est´an marcados en una l´ınea recta, y 11 puntos est´an marcados
sobre
otra l´ınea recta paralela a la primera. ¿Cu´antos:
a) tri´angulos
b) cuadril´ateros
hay con v´ertices en estos puntos?

Problema 2.58 Se tiene un conjunto de 15 palabras distintas. ¿De cu´antas maneras se puede
elegir
un subconjunto de no m´as de 5 palabras?
Problema 2.59 Hay 4 parejas casadas en un club. ¿De cu´antas maneras se puede elegir un
comit´e de 3 personas de tal manera que no haya un matrimonio incluido en el comit´e?
Problema 2.60 Hay 31 estudiantes en una clase, incluyendo a Pedro y a Juan. ¿Cu´antas
maneras
hay de elegir un equipo de futbol (11 jugadores), de tal manera que Pedro y Juan no est´en
juntos
en el equipo?
Problema 2.61 ¿De cu´antas maneras se pueden reacomodar las letras de la palabra
“HUMANOS”
de tal manera que tanto las vocales como las consonantes est´en en orden alfab´etico entre
s´ı? Ejemplo:
HAOMNUS (A-O-U, H-M-N-S).
Problema 2.62 Debemos elegir un equipo de 5 personas de entre 12 ni˜nas y 10 ni˜nos.
¿Cu´antas
maneras hay de hacerlo de tal forma que no haya m´as de 3 ni˜nos en el equipo?
Problema 2.63 ¿Cu´antas maneras hay de colocar 12 damas blancas y 12 damas negras en los
cuadros negros de un tablero de ajedrez?
Problema 2.64 a) ¿Cu´antas maneras hay de dividir a 15 personas en tres equipos de 5
personas
cada uno?
b) ¿Cu´antas maneras hay de elegir dos equipos de 5 personas cada uno de entre 15 personas?
Problema 2.65 ¿De cu´antas formas puedes elegir 10 cartas de una baraja de 52 cartas de
modo
que:
a) haya exactamente un as entre las cartas elegidas?
b) haya por lo menos un as entre las cartas elegidas?
Problema 2.66 ¿Cu´antos n´umeros de seis cifras tienen 3 d´ıgitos pares y 3 impares?

Problema 2.67 ¿Cu´antos n´umeros de 10 cifras tienen la suma de sus cifras igual a: a) 2; b) 3;
c)
4?
Problema 2.68 Una persona tiene 6 amigos. Cada noche, durante 5 d´ıas, invita a cenar a un
grupo
de 3 de ellos de modo que el mismo grupo no es invitado dos veces. ¿Cu´antas maneras hay de
hacer
esto?
Problema 2.69 Para participar en cierta loter´ıa en Rusia, uno debe elegir 6 n´umeros de entre
45
impresos en una tarjeta (todas las tarjetas son id´enticas).
a) ¿Cu´antas maneras de elegir los 6 n´umeros hay?
b) Despu´es de realizado el sorteo, los organizadores de la loter´ıa decidieron contar el
n´umero de maneras que hay de elegir los 6 n´umeros de tal manera que exactamente 3 de los
6 n´umeros elegidos
est´en entre los n´umeros de la combinaci´on ganadora. Ay´udalos a encontrar la respuesta.
DIFICIL El almacén de una universidad recibió 25 impresoras, de las cuales 10 son impresoras láser
y 15 son modelos de inyección de tinta. Si 6 de estas 25 se seleccionan al azar para que las
revise un técnico particular, ¿cuál es la probabilidad de que exactamente 3 de las seleccionadas
sean impresoras láser (de modo que las otras 3 sean de inyección de tinta)? (Devore 65)

EJERCICIOS COMBINATORIA (Curso-taller Básico Yucatán)

1. ¿Cuántas parejas diferentes compuestas por una mujer y un hombre se podrían formar a partir de 6 hombres y 5
mujeres?

2. ¿Cuántos tríos diferentes compuestos por un hombre, una mujer y un niño se pueden formar a partir de 4
hombres, 5 mujeres y 3 niños?

3. En una canasta hay 5 frutas diferentes y en otra canasta hay 3 verduras distintas. ¿De cuántas maneras se
puede elegir una fruta y una verdura?

4. ¿Cuántas palabras diferentes, con o sin significado, se pueden formar con las letras: A, L, E y C, sin que
ninguna letra se repita ni falte?

5. ¿Cuántas permutaciones simples (sin repetición) pueden hacerse con las letras de la palabra LEGAR?
a. ¿Cuántas de esas permutaciones comenzarán con una consonante?
b. ¿Cuántas comenzarán con una vocal?
c. ¿Cuántas comenzarán con la letra A?

6. Se tienen 10 bolitas de igual tamaño, 3 son de color rojo, 2 de color azul y 5 de color verde. ¿De
cuántas maneras diferentes se pueden ordenar en fila esas 10 bolitas?
a. ¿Cuántas de esas permutaciones comenzará con una bolita verde?
b. ¿Cuántas terminarán con una bolita roja?
c. ¿Cuántas comenzarán con una bolita azul y terminarán con una bolita verde?

7. ¿Cuántos números de 3 cifras diferentes pueden formarse con los dígitos: 1, 2, 3, 4 y 5?

8. ¿Cuántas palabras de 3 letras diferentes, con o sin significado, pueden formarse con las letras de la
palabra COMA?

9. Una empresa ferroviaria tiene 6 estaciones. ¿Cuántos tipos diferentes de boletos, donde se indique la
estación de salida y de llegada, deben imprimirse?

10. ¿Cuántos números de 3 cifras pueden formarse con los dígitos: 5, 6, 7, 8 y 9 (con repetición)?

11. ¿Cuántos números de 2 cifras pueden formarse con los diez dígitos, sin repetición?
12. ¿De cuántas maneras diferentes se puede elegir una comisión de 5 miembros a partir de 8 de personas?

a. Si una persona determinada debe estar siempre incluida


b. Si una persona determinada debe estar siempre excluida
c. Si una persona determinada debe estar siempre incluida y otra siempre excluida
d. Si dos personas determinadas nunca deben estar juntas en esa comisión

13. ¿Cuántas diagonales pueden trazarse en un polígono convexo de n lados?


14. ¿Cuántas comisiones diferentes, compuestas por 2 hombres y 3 mujeres, pueden formarse, a partir de
10 hombres y 12 mujeres?

15. ¿Cuántas palabras de 7 letras distintas ( 4 consonantes y 3 vocales ), con o sin significado, pueden
formarse a partir de 6 consonantes y 5 vocales, todas diferentes?

16. Calcular la probabilidad de, en una carrera de 12 caballos, acertar los 3 que quedan primeros (sin importar
cual de ellos queda primero, cual segundo y cual tercero).

17. Y si hubiera que acertar, no sólo los 3 caballos que ganan, sino el orden de su entrada en meta.

18. Se tienen 3 libros: uno de aritmética (A), uno de biología(B) y otro de cálculo(C), y se quiere ver de cuántas
maneras se pueden ordenar en un estante.

19. Se tienen 7 libros y solo 3 espacios en una biblioteca, y se quiere calcular de cuántas maneras se pueden
colocar 3 libros elegidos; entre los siete dados, suponiendo que no existan razones para preferir alguno.

20. ¿Cuántas permutaciones pueden formarse con las letras de la palabra BONDAD?

21. ¿De cuántas maneras se pueden ordenar las letras de la palabra AMASAS?

22. Un hospital cuenta con 21 cirujanos con los cuales hay que formar ternas para realizar guardias. ¿Cuántas
ternas se podrán formar?

23. ¿De cuántas maneras pueden entrar cuatro alumnos en tres aulas, si no se hace distinción de personas?

ETAPA ESTATAL
EJERCICIOS PROBABILIDAD Y ESTADÍSTICA PARA INGENIERÍA Y CIENCIAS MYERS PÁG. 47-48
2.21 A los participantes de una convención se les
ofrecen seis recorridos a sitios de interés cada uno de
los tres días. ¿De cuántas maneras se puede acomodar
una persona para ir a uno de los recorridos planeados
por la convención?
2.22 En un estudio médico los pacientes se clasifi can
en 8 formas de acuerdo con su tipo sanguíneo: AB+,
AB−, A+, A−, B+, B−, O− u O−; y también de acuerdo
con su presión sanguínea: baja, normal o alta. Encuentre
el número de formas en las que se puede clasifi car
a un paciente.
2.23 Si un experimento consiste en lanzar un dado
y después extraer una letra al azar del alfabeto inglés,
¿cuántos puntos habrá en el espacio muestral?
2.24 Los estudiantes de una universidad privada de
humanidades se clasifi can como estudiantes de primer
año, de segundo año, de penúltimo año o de último año,
y también de acuerdo con su género (hombres o mujeres).
Encuentre el número total de clasifi caciones posibles
para los estudiantes de esa universidad.
2.25 Cierto calzado se recibe en 5 diferentes estilos
y cada estilo está disponible en 4 colores distintos. Si
la tienda desea mostrar pares de estos zapatos que muestren
la totalidad de los diversos estilos y colores, ¿cuántos
diferentes pares tendría que mostrar?
2.26 Un estudio en California concluyó que al seguir
siete sencillas reglas para la salud, la vida de un hombre
se puede prolongar 11 años en promedio y la vida de
una mujer 7 años. Estas 7 reglas son: no fumar, hacer
ejercicio, uso moderado del alcohol, dormir siete
u ocho horas, mantener el peso adecuado, desayunar
y no ingerir alimentos entre comidas. De cuántas formas
puede una persona adoptar cinco de esas reglas
a seguir:
a) ¿Si la persona actualmente infringe las siete reglas?
b) ¿Si la persona nunca bebe y siempre desayuna?
2.27 Un urbanista de un nuevo fraccionamiento ofrece
a un futuro comprador de una casa la elección de
4 diseños, 3 diferentes sistemas de calefacción, un garaje
o cobertizo, y un patio o un porche cubierto. ¿De
cuántos planes diferentes dispone el comprador?
2.28 Un medicamento contra el asma se puede adquirir
de 5 diferentes laboratorios en forma de líquido,
comprimidos o cápsulas, todas en concentración normal
o alta. ¿De cuántas formas diferentes un doctor puede
recetar la medicina a un paciente que sufre de asma?
2.29 En un estudio económico de combustibles, cada
uno de 3 autos de carreras se prueba con 5 marcas
diferentes de gasolina en 7 lugares de prueba que se
localizan en diferentes regiones del país. Si se utilizan
2 pilotos en el estudio y las pruebas se realizan una vez
bajo cada uno de los distintos grupos de condiciones,
¿cuántas pruebas se necesitan?
2.30 ¿De cuántas formas distintas se puede responder
una prueba de falso-verdadero que consta de 9 preguntas?
2.31 Si una prueba de opción múltiple consiste en
5 preguntas, cada una con 4 respuestas posibles de las
cuales sólo 1 es correcta,
a) ¿de cuántas formas diferentes un estudiante puede
elegir una respuesta a cada pregunta?
b) ¿de cuántas maneras un estudiante puede elegir una
respuesta a cada pregunta y tener incorrectas todas
las respuestas?
2.32 a) ¿Cuántas permutaciones distintas se pueden
hacer con las letras de la palabra columna?
b) ¿Cuántas de estas permutaciones comienzan con la
letra m?
2.33 Un testigo de un accidente de tránsito, en el cual
huyó el culpable, dice a la policía que el número de la matrícula
contenía las letras RLH seguidas de 3 dígitos, cuyo
primer número es un 5. Si el testigo no puede recordar
los últimos 2 dígitos, pero tiene la certeza de que los 3
eran diferentes, encuentre el número máximo de matrículas
de automóvil que la policía tiene que verifi car.
2.34 a) ¿De cuántas maneras se pueden formar 6 personas
para abordar un autobús?
b) Si 3 personas específi cas, de las 6, insisten en estar
una después de la otra, ¿cuántas maneras son posibles?
c) Si 2 personas específi cas, de las 6, rehúsan seguir una
a la otra, ¿cuántas maneras son posibles?
2.35 Un contratista desea construir 9 casas, cada una
con diferente diseño. ¿De cuántas formas puede colocar
estas casas en una calle si hay 6 lotes en un lado de la
calle y 3 en el lado opuesto?
2.36 a) ¿Cuántos números de tres dígitos se pueden
formar con los dígitos 0, 1, 2, 3, 4, 5 y 6, si cada
dígito se puede usar sólo una vez?
b) ¿Cuántos de estos números son impares?
c) ¿Cuántos son mayores que 330?
2.37 ¿De cuántas maneras se pueden sentar 4 niños y
5 niñas en una fi la, si unos y otras se deben alternar?
2.38 Cuatro matrimonios compran 8 lugares en la
misma fi la para un concierto. ¿De cuántas maneras diferentes
se pueden sentar
a) sin restricciones?
b) si cada pareja se sienta junta?
c) si todos los hombres se sientan juntos a la derecha
de todas las mujeres?
2.39 En un concurso regional de ortografía, los 8 fi -
nalistas son 3 niños y 5 niñas. Encuentre el número
de puntos muestrales en el espacio muestral S para el
número de ordenamientos posibles al fi nal del concurso
para
a) los 8 fi nalistas;
b) las primeras 3 posiciones.
2.40 ¿De cuántas formas de pueden llenar las cinco
posiciones iniciales en un equipo de baloncesto con 8 jugadores
que pueden jugar cualquiera de las posiciones?
2.41 Encuentre el número de formas en que 6 profesores
se pueden asignar a 4 secciones de un curso
introductorio de psicología, si ningún profesor se asigna
a más de una sección.
2.42 Se sacan 3 billetes de lotería para el primer,
segundo y tercer premios de un grupo de 40 boletos.
Encuentre el número de puntos muestrales en S para
dar los 3 premios, si cada concursante sólo tiene un
billete.
2.43 ¿De cuántas maneras se pueden plantar 5 árboles
diferentes en un círculo?
2.44 ¿De cuántas formas se puede acomodar en círculo
una caravana de ocho carretas que proviene de Arizona?
2.45 ¿Cuántas permutaciones distintas se pueden hacer
con las letras de la palabra infi nito?
2.46 ¿De cuántas maneras se pueden colocar 3 robles,
4 pinos y 2 arces a lo largo de la línea divisoria de una
propiedad, si no se distingue entre árboles del mismo
tipo?
2.47 Una universidad participa en 12 juegos de fútbol
durante una temporada. ¿De cuántas formas puede
el equipo terminar la temporada con 7 ganados, 3 perdidos
y 2 empates?
2.48 Nueve personas se dirigen a esquiar en tres automóviles
que llevan 2, 4 y 5 pasajeros, respectivamente.
¿De cuántas maneras es posible transportar a las
9 personas hasta el albergue en todos los autos?
2.49 ¿Cuántas formas hay para seleccionar a 3 candidatos
de 8 recién graduados igualmente califi cados
para las vacantes de una empresa contable?
2.50 ¿Cuántas formas hay en que dos estudiantes no
tengan la misma fecha de cumpleaños en un grupo de
60?

RESPUESTAS IMPARES

2.21 18
2.23 156
2.25 20
2.27 48
2.29 210
2.31 a) 1024; b) 243
2.33 72
2.35 362,880
2.37 2,880
2.39 a) 40,320; b) 336
2.41 360
2.43 24
2.45 3,360
2.47 7,920
2.49 56

ETAPA NACIONAL DEVORE 65-67

Clase 1: Combinaciones y diagramas de árbol

1. Cuatro pueblos A, B, C, y D están comunicados por diferentes caminos. Del pueblo A al B


hay 2 caminos. De B a C hay 4 caminos. De C a D hay 5 caminos. Pablo vive en el pueblo
A y necesita ir a hacer negocios al pueblo D, teniendo que pasar por todos los pueblos.

a) ¿De cuántas maneras distintas puede realizar su viaje del pueblo A al D? ______________

b) ¿Bastaría con sumar los caminos entre los pueblos? ¿Por qué? ______________________

c) ¿Cómo calcularías la cantidad de caminos diferentes que hay para ir de A a D sin hacer un
diagrama de árbol ni contando de uno en uno? _____________________________________
d) ¿Qué operación aritmética utilizarías para calcular el total de rutas? __________________

e) Si se cierra uno de los cuatro caminos de B a C por mantenimiento, ¿cuántas rutas le


quedarían para ir de A a D? ____________________________________________________

f) Si se cerrarán de manera alternada cada uno de los caminos para darles mantenimiento, ¿cuál
de estos cierres afectaría más a los automovilistas al reducir a la mitad el número de rutas para
ir del pueblo A al pueblo D? ___________________________________________

El diagrama de árbol es una herramienta gráfica que permite explorar los posibles resultados de
un evento y facilita el cálculo de probabilidades.

1. En un restaurante se hicieron 2 diferentes sopas, 3 diferentes guisados y 2 diferentes


postres.
a. ¿De cuántas maneras diferentes puede escoger una sopa, un guisado y un postre?
__________________________________________________________
b. Y si además le dan a escoger entre 3 bebidas, ¿cuántas opciones tiene ahora?
________________________________________________________________

2. Juan tiene para entrenar atletismo, 3 sudaderas, 6 pants y 2 pares de tenis,


a. ¿cuántas combinaciones distintas puede hacer Juan para vestirse y entrenar
atletismo? _______________________________________________________
b. ¿Y si le regalaran 2 sudaderas más? ___________________________________

3. Pedro tiene 3 camisas de distintos colores, 2 pantalones y 2 pares de zapatos. Dibuja el


diagrama que representa correctamente de cuantas maneras distintas puede elegir su
vestuario.
a. ¿Y si los quisiera combinar con 4 sombreros?
________________________________________________________________

Las combinaciones hacen referencia a la cantidad de grupos diferentes que se pueden hacer
con los elementos de un conjunto, por ejemplo, ¿cuántos grupos de dos cifras se pueden hacer
con las cifras 1,2 y 3? En este caso la respuesta es tres grupos (1,2), (1,3) (2,3). Como puede
verse, lo que distingue a un grupo de otro son los elementos que lo forman, en este caso el
orden no es importante, ya que, por ejemplo, (1,2) y (2.1) es el mismo grupo.

Clase 2: Combinaciones y arreglos rectangulares

Organizados en equipos resuelvan los siguientes problemas.


1. Samuel vende arreglos florales y para esta semana ha conseguido las siguientes clases de
flores:

margarita rosa tulipán


lirio

Si en cada arreglo utiliza solamente dos tipos de flores, ¿cuántos arreglos diferentes podrá
elaborar? ________________________________________________________

2. En una nevería se venden los siguientes sabores: fresa, vainilla, limón, nuez y chocolate.
¿De cuántas formas diferentes se puede servir un helado de dos sabores distintos?
_____________________________________________________________

3. De los seis representantes de los grupos de primer grado, se va a formar una comisión de
tres alumnos que se entrevistará con el director para solicitarle una fiesta de fin de curso.
¿De cuántas formas diferentes se puede integrar la comisión?
_____________________________________________________________________

4. En un convivio hay 4 hombres y 3 mujeres y se quieren formar parejas para bailar. ¿Cuál
será el arreglo rectangular que nos sirve para saber el total de posibles parejas que se
pueden conformar? ________________________________________________

Diana Vero Ana

Raúl

Juan
José

Beto

5. En el segundo grado de secundaria hay cinco estudiantes que son buenos para jugar
básquetbol. El entrenador elegirá a sólo tres estudiantes para integrar una preselección de
la escuela. ¿De cuántas formas diferentes (combinaciones) puede elegir a los jugadores?
____________________________________________________________

6. ¿Cuántos grupos de dos cifras se pueden hacer con las cifras 1, 2 y 3?


a) Si las cifras de cada grupo son diferentes. _________________________________
b) Si las cifras de cada grupo pueden ser iguales. _____________________________

7. Considera que alguien desea viajar en autobús, tres o avión, para un periodo de
vacaciones de una semana, a uno de los cinco lugares de veraneo más concurridos del
país. ¿De cuántas maneras diferentes se podría lograr esto? _____________________

Clase 3: Variaciones

1. Se desea hacer una bandera de dos franjas y para ello se cuenta con cuatro colores de
tela: rojo, verde, azul y amarillo. Encuentra todas las posibles formas en las que se
pueden combinar los colores para elaborar la bandera.
a. ¿Cuántas posibilidades hay de elaborar la bandera si se repiten los colores?____
b. ¿Cuántas posibilidades hay de elaborar la bandera si no se repiten los colores?__
c. ¿Cuántas opciones tendremos si en las opciones añadimos el color violeta?_____

2. ¿Cuántas banderas diferentes de tres franjas, se pueden formar con


los colores rojo, azul, verde y blanco? Cada bandera debe tener tres
colores, uno en cada franja. _______________________

3. En una caja hay cinco fichas marcadas con los números 1, 3, 5, 7 y 9. Se extrae una ficha
de la caja y se anota su número. La ficha extraída se regresa a la caja y nuevamente se
realiza una extracción. ¿Cuántos números diferentes de dos cifras es posible formar?
___________________________________

4. ¿Cuántos números diferentes de dos cifras se pueden formar si la primera ficha que se
extrae no se regresa a la caja? ____________________________________________

5. Considerando nuevamente las cifras 1, 3, 5, 7 y 9, ¿cuántos números diferentes de tres,


cuatro y cinco cifras distintas es posible formar? _______________________
a. ¿Cuántos números diferentes se pueden colocar en el primer nivel (centenas)?
b. ¿Cuántos números diferentes se pueden colocar en el segundo nivel (decenas)?
c. ¿Cuántos números diferentes se pueden colocar en el tercer nivel (unidades)?
6. Con las cifras 0, 1, 2, 3, 4 y 5.
a. ¿Cuántos números diferentes de tres cifras sin repetir se pueden formar?______
b. De los anteriores, ¿cuántos son pares? _______________________
c. Si se ordenan de mayor a menor, ¿qué lugar ocupa el 234? ________________

7. En un edificio nuevo hay 5 departamentos, cada departamento cuenta con un lugar de


estacionamiento. Se han habitado dos departamentos, únicamente, el de Carmen y el de
Daniel, quienes pueden colocar cada noche sus coches en el lugar que prefieran, si no
está ocupado. ¿De cuántas formas diferentes pueden estacionarse? Represéntenlo en un
diagrama de árbol.

8. Ha llegado un nuevo vecino,


a. ¿de cuántas maneras distintas pueden estacionar los coches los tres vecinos?___
b. ¿Resultan más o menos maneras que en el caso anterior? __________________
c. ¿Cuántas maneras habrá de estacionarse cuando todos los departamentos estén
ocupados, si todos los vecinos tienen coche? _______________________

9. ¿Cuántas palabras de tres letras se pueden formar con cinco consonantes y tres vocales
de modo que cada palabra comience y termine en consonante?_____________

10.El ácido ribonucleico de nuestras células contiene 4 tipos de bases componentes: la


adenina (A), el uracilo (U), la citosina (C) y la guanina (G), las que al combinarse de 3 en
3, van formando una cadena de aminoácidos que dan lugar a las proteínas. ¿Cuántas
combinaciones diferentes puede hacerse al seleccionar 3 de las 4 bases, sin que se repitan
los componentes? _______________________ Tema de investigación.

En el caso de las variaciones, lo que distingue a un subgrupo de otro son los elementos que lo
forman o el orden, por tanto, en el ejemplo anterior el número de variaciones es seis, ya que
cada uno de los tres subgrupos se duplica al cambiar el orden de sus elementos.

Clase 4: Permutaciones

1. ¿Cuáles son todos los posibles resultados al lanzar tres monedas? ¿Cuántos resultados se
obtienen al lanzar cuatro monedas?

2. Para un espectáculo, un mago se viste con sombrero, camisa,


pantalón y zapatos. En su baúl lleva 5 sombreros, 5 camisas,
5 pantalones y 5 pares de zapatos. Cada prenda es de uno de
estos colores: rojo, negro, amarillo, verde y azul y de cada
tipo de prenda tiene exactamente una de cada color. Si no
puede usar dos prendas del mismo color y no puede usar simultáneamente rojo y negro,
¿de cuántas maneras se puede vestir el mago para el
espectáculo?_______________________

3. Andrea, Caro y Daniela se citan en una cafetería. Las tres amigas


llegaron a la cita de una en una. Determinar todos los
ordenamientos posibles en que pudieron haber llegado.
______________________________________________

4. Andrea, Bety, Caro y Daniela se citan en una cafetería. Las cuatro amigas llegaron a la
cita de una en una. Determinar todos los ordenamientos posibles en que pudieron haber
llegado. ______________________________________________

5. Si Caro es la amiga que llegó primero, determina todos los ordenamientos posibles en que
pudieron haber llegado las otras tres amigas. _______________________

6. En el grupo de segundo de secundaria había un gran problema por los casilleros


desordenados, así que el profesor decide formar cuatro equipos para acomodar los
casilleros durante un mes. ¡Ningún equipo quiere empezar! ¿De cuántas formas se puede
ordenar los equipos? _______________________

7. ¿Cuántos números de cuatro cifras diferentes se pueden formar con las cifras 2, 3, 5 y 7?
Con las mismas cifras, ¿cuántos números de cuatro cifras se podrían formar pudiendo
repetir cifras en un mismo número? _______________________

8. Al final del curso escolar se organizará la escolta de la escuela “Vicente Guerrero”, para
ello se eligió a seis alumnos de segundo grado.
a) ¿De cuántas formas diferentes pueden colocarse los alumnos en la escolta?_______
b) Si la abanderada es Mariana porque tuvo el promedio más alto, ¿de cuántas formas
pueden colocarse en la escolta los demás integrantes sin cambiar dicha
posición?___________________________________________________________
c) Juan tiene un volumen de voz fuerte, por lo que se decide ponerlo de sargento. Si
Mariana es la abanderada y Juan el sargento, ¿de cuántas maneras diferentes pueden
colocarse los otros cuatro integrantes? _____________________________

9. Determina el número de enteros de seis dígitos (que no comiencen con cero) en los que
ninguno se pueda repetir. _______________________

En el caso de las permutaciones entre los elementos de un conjunto n, intervienen todos los
elementos del conjunto. Por ejemplo, ¿cuántos números diferentes de cuatro cifras se pueden
formar con las cifras 1, 2, 3, 4? En este caso la respuesta es 24
V. FALTA SEPARADORES (¿SERÁ IMPORTANTE?)

VI. PRINCIPIO DE INCLUSIÓN Y EXCLUSIÓN

Si un proceso de selección se puede realizar de dos formas de modo que la primera admite n
opciones, la segunda admite m opciones y hay r opciones comunes a ambas, entonces el
número total de selecciones posibles es n+m-r.

El principio de inclusi´on y exclusi´on es una especie de generalizaci´on de la t´ecnica del


complemento
para resolver problemas de combinatoria. Enunciarlo formalmente es complicado y muy poco
ilustrativo, as´ı que mejor resolveremos algunos problemas relacionados para familiarizarnos
con ´el.
Problema 2.93 En un instituto de investigaci´on cient´ıfica trabajan 67 personas. De ´estas, 47
hablan ingl´es, 35 alem´an y 23 ambos idiomas. ¿Cu´antas personas en el instituto no hablan ni
ingl´es
ni alem´an?
Problema 2.94 Para un n´umero natural n se define '(n) (la funci´on de Euler) como la cantidad
de n´umeros menores o iguales a n que son primos relativos con n. Calcula: '(10), '(120), '(354)
y
'(1080).
Problema 2.95 ¿Cu´antas permutaciones distintas pueden efectuarse con n elementos en las que
dos de ellos, a y b, no est´en juntos? ¿Y en las que no lo est´en tres, a, b y c (en cualquier
orden)?
¿Y en las que ning´un par de los elementos a, b y c est´e junto?
Problema 2.96 Un encuadernador debe encuadernar 12 libros diferentes en rojo, verde y azul.
¿De cu´antos modos puede hacerlo, si por lo menos un libro debe estar encuadernado en cada
color?
Problema 2.97 En un centro de investigaci´on en matem´aticas trabajan varias personas, y cada
una de ellas habla por lo menos una lengua extranjera. Seis hablan ingl´es; seis, alem´an;
siete, franc´es.
Cuatro hablan ingl´es y alem´an; tres, alem´an y franc´es; dos, franc´es e ingl´es. Una
persona habla los
tres idiomas. ¿Cu´antas personas trabajan en el centro? ¿Cu´antas de ellas hablan solamente
ingl´es?
¿Y solamente franc´es?
Problema 2.98 ¿De cu´antos modos se pueden permutar las letras de la palabra “tictac” si dos
letras iguales no pueden ir una a continuaci´on de la otra? Lo mismo, pero para la palabra
“tamtam”.
Problema 2.99 ¿Cu´antos n´umeros enteros no negativos, menores que un mill´on, contienen a
todas
las cifras 1, 2, 3 y 4? ¿Cu´antos est´an formados solamente por estas cifras?
Problema 2.100 ¿Cu´antos n´umeros de seis cifras se pueden escribir a partir de las cifras del
n´umero 1233145254, de forma que no haya dos cifras iguales juntas?
Problema 2.101 ¿De cu´antos modos se pueden permutar las cifras del n´umero 12341234, de
forma
que no haya dos cifras iguales juntas? El mismo problema para el n´umero 12345254.
Problema 2.102 ¿De cu´antas maneras se pueden permutar las cifras del n´umero 1234114546,
de
forma que no haya tres cifras iguales juntas? ¿Y de modo que no haya dos cifras iguales juntas?
Problema 2.103 A un ascensor subieron 8 personas. ¿De cu´antas maneras pueden bajarse en
cuatro pisos, de modo que en cada piso salga por lo menos una persona?

Problema 2.104 Demostrar que r cosas diferentes se pueden distribuir entre n + p personas, de
modo que n dadas obtengan por lo menos un objeto, de

formas.
Problema 2.105 ¿De cu´antas formas se pueden sentar juntos 3 ingleses, 3 franceses y 3 turcos,
de
modo que no haya tres compatriotas juntos? El mismo problema, pero con la condici´on de que
no
haya dos compatriotas juntos.
Problema 2.106 ¿De cu´antas formas se pueden sentar a una mesa redonda 3 ingleses, 3
franceses
y 3 turcos, de modo que no haya dos compatriotas juntos?
Problema 2.107 ¿De cu´antas maneras pueden seis personas escoger de entre seis pares de
zapatos
uno derecho y uno izquierdo, de modo que ninguno obtenga un par? Lo mismo para 9 pares y 6
personas.
Problema 2.108 Las casillas de un tablero de ajedrez se pintan de 8 colores, de modo que en
cada
fila horizontal se encuentren los 8 colores, y en cada fila vertical no se encuentren dos casillas
juntas
pintadas del mismo color. ¿De cu´antas formas es posible hacer esto?
El Principio del palomar, de los casilleros o de Dirichet
Si disponemos de n cajas para colocar n+k bolas hemos de colocar más de una bola en alguna
caja.
Si se tiene un conjunto n de objetos, repartidos en m casilleros y n>km, con K un número
natural, entonces hay al menos un casillero donde hay (k+1) o más objetos.
Ejemplo:
Entre trece personas hay al menos dos que nacieron el mismo mes.

Principio del complementario


El número de elementos de un conjunto finito que verifican una propiedad es el número de
elementos de dicho conjunto menos el número de elementos que no la verifican
Ejemplo: Determinar el número de resultados que se pueden obtener al tirar un dado rojo y uno
verde cuya suma es menor que 11.

Número Catalán
En combinatoria, los números de Catalán forman una secuencia de números naturales que
aparece en varios problemas de conteo que habitualmente son recursivos. Obtienen su nombre
del matemático belga Eugène Charles Catalán (1814–1894).

El n-ésimo número de catalán se obtiene, aplicando coeficientes binomiales, a partir de la


siguiente fórmula:
Número Bell
Es el número de particiones (formas de re arreglar) en que puedes dividir un conjunto en
subconjuntos no vacíos, teniendo todos los elementos del conjunto original. Si en un conjunto
tienes solo 3 elementos A B C puedes Tendrás las siguientes particiones es : 1.- ( A) ( B) (C) 2.-
( A B) ( C) 3.- ( A C ) ( B ) 4.- ( B C ) ( A ) 5.- ( A B C )

Cada paréntesis indica un subconjunto. Por lo que el número de Bell para 3 elementos es 5
En combinatoria, el n-ésimo número de Bell, llamado así por Eric Temple Bell, es el número de
particiones de un conjunto de n elementos, o equivalentemente, el número de relaciones de
equivalencia en el mismo. Comenzando con B0 = B1 = 1, los primeros números de Bell son:
1, 1, 2, 5, 15, 52, 203, 877, 4140, 21147, 115975, …

PROBLEMARIO ENTRENAMIENTO JALISCO

1. Se tienen 6 sabores diferentes de helados. Ernesto quiere comprar helado con dos bolas y
quiere saber cuántas posibles combinaciones puede hacer. (P4 2010 JALISCO)
2. Una caja contiene 20 pelotas amarillas. 9 rojas y 6 azules. Si las pelotas son seleccionadas
al azar, ¿cuál es el menor número de pelotas que necesitas sacar de la caja para asegurar
que tienes al menos dos pelotas de cada color? (P24 2010 JALISCO)

3. Colorea seis de los diez círculos del dibujo, de manera que siempre haya dos círculos
coloreados en cada recta. (P32 2010 JALISCO)

4. El profesor Gerardo tiene 2 sacos, 3 pantalones y 4 corbatas, todos distintos. ¿De cuántas
formas diferentes se puede vestir? (P35 2010 JALISCO)

5. Forma palabras con las siguientes reglas: la primera palabra es a; para armar una nueva
palabra cada vez que tengas una a la cambias por una b y cada que tengas una b la
cambias por una ba. Siguiendo estas reglas las primeras palabras son: a, b, ba, bab,
babba. ¿Qué palabra sigue? (P45 2010 JALISCO)

6. Los números del 1 al 9 se colocaron en los 5 anillos olímpicos de tal forma que la suma en
cada anillo es 11. ¿Los puedes colocar de tal forma que la suma en cada anillo sea 14?
(P50 2010 JALISCO)

7. ¿Cuál es el mínimo numero de cuadritos que tienes que rellenar para que tanto m como
m’ sean rectas de simetría del cuadrado? (P52 2010 JALISCO)
8. Cinco niños juegan a las escondidas en el patio de su escuela, cuatro se esconden y otro
los busca. En ese patio hay sólo 3 escondites, los que diario usan: atrás del árbol, atrás
del bote de basura y bajo la banca (en donde caben dos niños), les toca esconderse a
Ana, Beto, Carlos y David. ¿De cuántas formas distintas se pueden repartir en los
escondites? (P4 2011 JALISCO)

9. La abuela guarda bolsitas de té en una caja con 6 casillas como la que muestra la figura.
Tiene 6 variedades de te: Negro, Verde, Manzanilla, Hierbabuena, Canela y Limón. Pone
cada variedad en una casilla, y nunca pone el Negro y el Verde en las casillas de en medio
ni en casillas vecinas. ¿De cuántas maneras distintas puede acomodar las 6 variedades de
té en la caja? (P8 2011 JALISCO)

10.¿Cuántos triángulos isósceles distintos se pueden formar, de tal manera que las
longitudes de sus lados sean números enteros y su perímetro sea 25? (P10 2011
JALISCO)

11.Alberto está entrenando para un campeonato de ciclismo. Cada mañana sale de su pueblo
tomando uno de los 7 caminos secundarios que hay hacia la carretera principal. Los
caminos secundarios son de doble sentido (puede ir y regresar por ellos). El camino
principal es de un solo sentido (sólo puede avanzar en una dirección). Una vez en el
camino principal recorre una parte y regresa por un camino secundario, diferente al que
tomó al inicio. Si una ruta consiste en salir del pueblo por un camino secundario, recorrer
parte del camino principal y regresar por un camino secundario diferente ¿Cuántas rutas
diferentes puede hacer Alberto? (P12 2011 JALISCO)
12.Una línea de camiones ha decidido premiar con pasaje gratis a todos las personas que la
suma de las cifras del número que aparece en su boleto de camión sea 21. La promoción
durará sólo por el mes de Marzo, así que mandaron imprimir boletos que van del 1 al
2000. ¿Cuántos boletos de éstos darán pasaje gratis a los usuarios? (P23 2011 JALISCO)

13.Una caja contiene 20 pelotas amarillas, 9 rojas y 6 azules. Si las pelotas son
seleccionadas al azar, ¿cuál es el menor número de pelotas que necesitas sacar de la caja
para asegurar que tienes al menos dos pelotas de cada color? (P29 2011 JALISCO)

14.Mónica escribe todos los números de dos cifras en los cuales, la suma de los dos dígitos
que forman el número es 8. Luego suma todos los números que escribió. ¿Cuál es el
resultado que obtiene Mónica? (P2 2012 JALISCO)

15.En un edificio se numeraron todas las puertas de las oficinas utilizando placas que
contenían un dígito cada una (por ejemplo, al numerar la 14ª puerta se usaron dos
placas, una con el número 1 y otra con el 4). Si se utilizaron 35 placas, ¿cuántas puertas
hay? (P9 2012 JALISCO)

16.A Rosa le gusta calcular la suma de los dígitos que ve en su reloj digital (por ejemplo, si el
reloj marca las 21: 17 Rosa obtiene 11). ¿Cuál es la máxima suma que puede obtener?
(P10 2012 JALISCO)

17.En la figura se tiene que llegar del círculo A al círculo B siguiendo las flechas. En cada
camino se calcula la suma de los números por los cuales se pasó. ¿Cuántas sumas
diferentes se pueden obtener? (P14 2012 JALISCO)
18.Quiero escribir en el pizarrón una palabra de 5 letras, una de 6 letras, una de 7 letras y
así sucesivamente. ¿Cuántas letras escribiré para formar las primeras 9 palabras? (P5
2013 JALISCO)

19.En el dibujo se muestran los únicos rectángulos de lados enteros que tienen área igual a 4
cm. El área de un rectángulo se obtiene de multiplicar uno de los lados verticales por uno
de los lados horizontales. Haz una lista de todas las posibles medidas de los rectángulos
de lados enteros que tengan área igual a 48 cm2. (P20 2013 JALISCO)

20.El cuerpo de un gusano está formado por 6 círculos. ¿De cuantas formas diferentes se
puede colorear si 3 de los círculos deben ser blancos y 3 deben ser grises? La figura
muestra el ejemplo de una de las posibilidades, encuentra todas. (P23 2013 JALISCO)

21.Jorge tiene 5 dulces de caramelo de distintos colores: Amarillo, Verde, Rojo, Morado y
Blanco; y tiene también 4 chiclosos de diferentes sabores: Cajeta, Fresa, Limón y
Naranja. Quiere llevar a la escuela 2 dulces y 3 chiclosos. ¿De cuantas maneras puede
hacer su selección? (P30 2013 JALISCO)

22.En una competencia de barcos de vela hay 86 tripulantes en todos los barcos. Los barcos
de una sola vela tienen tripulaciones de cinco personas y los de dos velas de siete
personas. Indica todas las posibles combinaciones de barcos que puede haber. (P33 2013
JALISCO)

23.¿Cuál posición de las manecillas del reloj determina el menor ángulo, a las 8:20, a las
12:20 o a las 13:30 horas? Explica por qué. (P40 2013 JALISCO)

24.Hugo dobla una hoja de papel cinco veces. Luego hace un agujero en el papel doblado
como se muestra en la figura, y desdobla el papel. ¿Cuántos agujeros aparecen en el
papel desdoblado? (P44 2013 JALISCO)

25.En una tienda venden tres artículos de $ 30.00 cada uno y dos artículos de $ 40.00 cada
uno. Encuentra todas las cantidades diferentes de dinero que puede ganar la tienda con la
venta de uno, varios o todos los artículos. (P47 2013 JALISCO)

26.El señor Ye y la señora Zeta quieren nombrar a su bebe de manera que las iniciales de sus
dos nombres estén en orden alfabético y que no tengan letras repetidas. ¿De cuantas
maneras se puede hacer esto? Supongamos que no hay nombres que empiecen con n, por
tanto, se consideran 26 letras. (P52 2013 JALISCO)

COMPENDIO

1. ¿Cuántos números de dos dígitos (es decir, números mayores que 9 y menores que 100)
cumplen que la suma de sus dígitos es par?

2. ¿Cuántos resultados diferentes podemos obtener sumando dos números distintos del conjunto
1,2,3,...,10?

3. ¿Cuántos números de tres dígitos podemos formar si: a) el primer dígito es par, el segundo
dígito es mayor que siete y el tercero es impar?, b) el primer dígito es múltiplo de tres, el
segundo es menos o igual que cinco y el tercero es cero?

4. ¿Cuántas palabras de tres letras se pueden formar si se dispone de un alfabeto con dos
letras: a y b? (Nota: Son permisibles palabras como bba.)

5. José, Juan y 8 personas más compiten en una carrera de velocidad, sin considerar la
posibilidad de empates, ¿en cuántos órdenes distintos pueden llegar a la meta los 10 corredores,
con José y Juan en las dos primeras posiciones?

6. De la ciudad A a la ciudad B conducen 2 caminos, de la ciudad B a la ciudad C conducen tres


caminos. Entonces, de la ciudad A a la C, pasando por B, ¿cuántos caminos hay?
7. ¿Cuántas placas distintas hay con dos letras a la izquierda y tres números a la derecha?
(Nota: Consideraremos el alfabeto de 27 letras castellanas.)

8. A la cumbre de una montaña conducen cinco caminos. ¿De cuántas maneras puede trepar un
turista a la montaña y descender por ella si no puede usar dos veces el mismo camino?

9. Un cubo de madera se pinta de rojo y después se parte en 27 cubitos. ¿Cuántos cubitos


tienen exactamente: a) 2 caras rojas? b) 0 caras rojas?

10. A un cubo de 4x4x4 se le pintan sus cara de rojo, después se divide en 64 cubitos de 1x1x1.
¿Cuántos de los cubitos de 1x1x1 tienen pintadas 1 o 2 caras exactamente?

11. Cinco cartas fueron enviadas a cinco destinos distintos, colocándolas aleatoriamente en los
cinco sobres con las direcciones ya escritas. ¿De cuántas maneras distintas pudieron enviarse las
cinco cartas de tal manera que al menos una de ellas no llegue a su destino correcto?

12. Una diseñadora dispone de 5 tonos de naranja, 7 tonos de verde y 4 tonos de morado, y
quiere escoger dos de estos para un logotipo. Ella considera que usar dos tonos del mismo color
es aburrido, pero todas las demás combinaciones le agradan. ¿Cuántas opciones tiene?

13. En la siguiente figura, cuántas rutas distintas puede seguir una hormiga para ir de la esquina
A a la esquina B, si sólo puede caminar sobre las líneas negras, y sólo se puede mover hacia
arriba y hacia la derecha.

14. El pequeño Luis tiene que hacer una figura que consiste de siete columnas, la primera tiene
un solo cuadrito, la segunda tiene dos cuadritos, y así, hasta la última que tiene 7 cuadritos.
¿Cuántos palillos necesita Luis, si cada cuadrito se forma con 4 palillos? Nota: dos cuadritos
comparten un mismo palillo si tienen un lado en común. A continuación se muestra la figura
hasta la tercera columna.
15. De entre los números de tres dígitos (000, 001, …, 999), ¿cuántos hay que el dígito central
sea el mayor?

16. ¿Cuántas parejas de enteros positivos impares tienen como suma 2006?

17. Un número que se lee igual de derecha a izquierda que de izquierda a derecha se dice que
es un número capicúa, por ejemplo 1221 y 3625263. ¿Cuántos números capicúas de 6 dígitos
existen? (Nota: los números de 6 dígitos son los del 100 000 al 999 999.)

18. Un club de tenis desea organizar un campeonato de individuales entre sus 1023 socios. Para
ellos acuerdan las siguientes condiciones: Cada jugador que pierda un partido queda
automáticamente eliminado del torneo. Cada partido debe jugarse con una pelota nueva y
organizar el torneo de tal manera que haya la menor cantidad de pelotas. ¿Cuál será la cantidad
mínima de pelotas que deben comprarse para efectuar dicho torneo?

19. En una feria hay tres juegos: las cataratas, la montaña rusa y la rueda de la fortuna. A la
feria asistió un grupo de 39 personas, de las cuales 5 decidieron no subirse a los juegos. De las
personas del grupo, 19 se subieron a las cataratas, 17 a la montaña rusa y 14 a la rueda de la
fortuna. De las personas del grupo que se subieron a dos o más juegos distintos; 8 se subieron a
las cataratas y a la montaña rusa, 4 a la montaña rusa y a la rueda de la fortuna, y 7 a las
cataratas y a la rueda de la fortuna. a) ¿Cuántas personas del grupo se subieron a los tres
juegos?, b) ¿Cuántas personas del grupo se subieron solamente a un juego?

20. En un monte hay varias fincas, cada una de las cuelas está unida a las restantes por un
camino. Si sabemos que hay 36 caminos, ¿cuál es el número de fincas?

21. Un costal está lleno de canicas de 20 colores distintos. Al azar se van sacando canicas del
costal. ¿cuál es el mínimo número de canicas que deben sacarse para poder garantizar que en la
colección tomada habrá al menos 100 canicas del mismo color?

22. ¿Cuántos números de tres cifras hay tales que la suma de las primeras dos cifras es igual a
la tercera cifra?

23. ¿Cuántos números múltiplos de 6 menores que 1000 tienen la propiedad de que la suma de
sus cifras es 21?

24. ¿Cuántas cantidades diferentes de dinero puedes pagar con cambio exacto si tienes dos
monedas de 1 peso y 2 monedas de 50 centavos?
25. Pablito recorta cada una de las cifras del número 2003 de un periódico, y se pone a pegar
algunos de estos cuatro trocitos de papel (o tal vez todos) en un renglón de su cuaderno para
formar un número. ¿Cuántos números distintos puede construir de esta manera?

26. En una fiesta cada persona saludo exactamente a otras 3 personas. Si hubo en total 123
saludos (apretones de mano). ¿cuántas personas asistieron a la fiesta?

27. ¿Cuál es el mayor número de partes en que podemos dividir un círculo utilizando únicamente
3 líneas rectas?

28. ¿Cuál es el mínimo número de colores que necesitas para pintar un cubo de tal forma que
dos caras adyacentes no tengan el mismo color?

29. En un grupo con 13 personas, ¿podemos asegurar que hay dos que nacieron el mismo mes?
¿y si el grupo fuera de solamente 12 personas?

30. Una caja contiene 20 pelotas amarillas, 9 rojas y 6 azules. Si las pelotas son seleccionadas al
azar, ¿cuál es el menor número de pelotas que necesitas sacar de la caja para asegurar que
tienes al menos dos pelotas de cada color?

31. ¿Cuántos enteros positivos menores que 100 tienen un número impar de divisores positivos
distintos?

32. ¿Cuántos cuadrados hay en una cuadrícula de 3 x 3? ¿Y en una de 4 x 4?

Cinco niños juegan a las escondidas en el patio de su escuela, cuatro se esconden y otro los
busca. En ese patio hay sólo 3 escondites, los que diario usan: atrás del árbol, atrás del bote de
basura y bajo la banca (en donde caben dos niños), Les toca esconderse a Ana, Beto, Carlos y
David. ¿De cuántas formas distintas se pueden repartir en los escondites?

Un costal está lleno de canicas de 20 colores distintos. Al azar se van sacando canicas del costal.
¿Cuál es el mínimo número de canicas que deben sacarse para poder garantizar que en la
colección tomada habrá al menos 100 canicas del mismo color? (en el costal hay más de 100
canicas de cada uno de los 20 colores).

Cuatro parejas de novios se reúnen para ir a un concurso de baile. Sabiendo que: • Beatriz bailó
con Eduardo. • Alicia bailó con el novio de Clara. • Federico bailó con la novia de Gustavo. •
Daniela bailó con el novio de Alicia. • Gustavo bailó con la novia de Eduardo. (a) ¿Con quién
bailó Humberto? (b) Encuentra quiénes son parejas de novios.

Un arqueólogo ha descubierto que una antigua civilización usaba 5 símbolos para representar los

números: . Estos símbolos corresponden en algún orden a los dígitos 0, 1, 2, 3 y 4.

De este modo, cuando escriben representan un número en base 5:

El arqueólogo sabe que los siguientes tres números son consecutivos, ordenados de menor a
mayor:

, y .

Hallar el valor de cada símbolo y cuáles son los tres números consecutivos. P26 OEJ 2011

La abuela guarda bolsitas de té en una caja con 6 casillas como la que muestra la figura:

Tiene 6 variedades de té: Negro, Verde, Manzanilla, Hierbabuena, Canela y Limón. Pone cada
variedad en una casilla, y nunca pone el Negro y el Verde en las casillas de en medio ni en
casillas vecinas. ¿De cuántas maneras distintas puede acomodar las 6 variedades de té en la
caja? P27 OEJ 2011

Se escriben los dígitos 1, 2, 3, 4 en cuatro papelitos que se guardan en una caja. Si dos de los
papelitos se extraen al azar, ¿cuál es la probabilidad de que la suma de los números sea múltiplo
de 3?

Julia tiene en su ropero 3 faldas de colores blanco, rosado y marrón; 3 blusas de colores negro,
amarillo y azul, y 2 pares de zapatos de color marrón y negro. Combinando todas estas prendas
de vestir, ¿de cuántas maneras se puede vestir Julia?

En la adición, A y B representan dígitos o cifras diferentes de un número. ¿Cuántas adiciones


distintas dan el resultado 165?
En un torneo de fútbol cada equipo juega 19 partidos en total. Cada vez que un equipo gana
obtiene 3 puntos y si empata obtiene 1 punto. Al final del torneo el equipo de México obtuvo 28
puntos. ¿Cuál es el mínimo número de partidos que ganó?

Con las cifras 1, 3, 4, 5 y 6, ¿cuántos números de cuatro cifras distintas se podrán formar de
modo que acaben en cifra par?

Para formar un equipo de pádel se necesitan 4 jugadores y un entrenador, que se deben elegir
de entre un grupo de 10 jugadores y 3 entrenadores. ¿Cuántos equipos distintos se pueden
formar?

Los 13 alumnos de un grupo de 2º de Bachillerato desean que les hagan una foto a todos juntos,
en fila, como recuerdo de su paso por el instituto. En dicha foto no deben aparecer ni dos chicas
ni dos chicos juntos. Sabiendo que hay 7 chicas, ¿de cuántas formas distintas pueden colocarse?

Halla el número de capicúas de seis cifras.

Disponemos de 8 colores para pintar un mural dividido en 3 columnas; cada una de ellas se ha
de pintar de un color distinto. ¿Cuántos murales se pueden confeccionar incluyendo el color
verde siempre? ¿Y si quisiéramos que apareciera el azul pero no el negro?

Ocho ciclistas van por el carril bici en fila. ¿De cuántas formas pueden ir ordenados?

A una familia de 6 personas les ha tocado un viaje para dos personas. ¿De cuántas formas se
pueden repartir el viaje?

En un concurso de radio participan 7 personas, de las cuales, 2 pueden conseguir los premios,
que son: una enciclopedia y una radio. Sabiendo que una persona no puede conseguir los dos
premios, ¿cuántas posibles distribuciones hay?
Para hacer una transferencia bancaria, Marta tiene que teclear una clave de acceso que consta
de 8 cifras con los dígitos 0 y 1. ¿Cuántas claves distintas puede formar?

Para desayunar, Mario elige 4 pastas distintas de las 12 clases que tiene. ¿Cuántas posibles
elecciones hay?

Con las cifras impares, ¿cuántos números de tres cifras se pueden formar pudiéndose repetir las
cifras?

Cierto equipo de baloncesto cuenta con 11 jugadores, pero solo se necesitan 5 para jugar un
partido. ¿Cuántas alineaciones distintas se podrán formar?

Con todas las letras de la palabra TIJERA, ¿cuántas palabras, con o sin sentido, se pueden
formar sin repetir las letras?

En un campeonato de motos hay 15 participantes y tres premios a repartir. ¿De cuántas formas
se pueden repartir?

Belén necesita seleccionar 4 personas, entre los 20 candidatos que tiene, para formar su equipo
de trabajo. ¿De cuántas maneras puede hacer la selección?

¿Cuántos números de cuatro cifras distintas podemos formar con los dígitos 2, 4, 6, 8 y 9?

Con las letras de la palabra JUNIO, ¿cuántas palabras, con o sin significado, podemos formar con
4 letras, pudiendo estas repetirse?

En un torneo de balonmano hay 8 equipos participantes y solo 3 trofeos, ¿de cuántas maneras
distintas se pueden repartir los premios 1º, 2º y 3º?

Tenemos que formar un código de 6 cifras con los dígitos 0 y 1. ¿Cuántas posibilidades hay?
Sabiendo que los puestos de delegado y de subdelegado no pueden ser cubiertos por la misma
persona, calcula cuántas posibilidades hay para cubrir ambos cargos en una clase de 22
alumnos.

Con las letras de la palabra CUADERNO, ¿cuántas palabras, con o sin sentido, se pueden formar?

En una carrera organizada en un centro escolar participan los 6 finalistas de 3º DE SECUNDARIA


¿De cuántas formas distintas pueden llegar a la meta?

Con los dígitos impares, ¿cuántos números de cinco cifras distintas se pueden formar?

¿De cuántas formas se pueden repartir 4 bocadillos distintos entre 4 amigos, si cada uno debe
recibir solo uno?

Ana, Pilar y Susana tienen que elegir una optativa entre 3 posibilidades para el próximo curso. Si
entre ellas no quieren coincidir en la misma optativa, ¿de cuántas formas se podrían llegar a
repartir las optativas?

Marcos tiene 8 sabores distintos de helado para preparar copas de 3 sabores. ¿Cuántas copas
distintas puede preparar?

En una empresa se quieren contratar 5 agentes de seguridad. Si al proceso de selección se


presentan 10 personas, ¿de cuántas formas distintas se pueden ocupar las cinco plazas?

Un club de tenis dispone de 15 jugadores profesionales de los cuales debe seleccionar 8 para
jugar un torneo. ¿Cuántos grupos se pueden formar?

En un centro de trabajo se tienen que elegir a cuatro de sus 18 empleados para representar a la
empresa en una reunión del sector. ¿Cuántas elecciones diferentes pueden darse?

De una lista de 12 discos, Rosa tiene que seleccionar 5 diferentes para regalar. ¿Cuántas
selecciones distintas puede hacer?
Con 0, 1, 2, 3 y 4, ¿cuántos números de cinco cifras se pueden formar, sin repetir ningún dígito?

¿Cuántas ordenaciones pueden hacerse con las letras de la palabra PINCEL de modo que
comiencen y terminen por consonante?

Para formar la tripulación de un avión se eligen 3 comandantes y 4 azafatas entre un grupo de


11 personas, 5 de las cuales son comandantes y el resto, azafatas. ¿Cuántas tripulaciones
distintas se pueden formar?

El sistema actual de matrículas combina 4 cifras con 3 letras, que se eligen entre 10 cifras y 26
letras. ¿Cuántas matrículas distintas se pueden hacer?

¿De cuántas formas pueden sentarse 4 hombres y 4 mujeres en una fila de un cine si quieren
estar alternados?

Dos amigos juegan al futbolín y acuerdan que será vencedor el que gane dos partidas seguidas o
tres alternativas ?no hay empate?. ¿De cuántas formas puede desarrollarse el juego?

Tengo dos monedas de 1 €, dos de 2 € y dos de 50 cent. Tomando tres de las seis monedas,
¿cuántas sumas distintas puedo hacer?

¿Cuántos productos de tres cifras iguales o distintas podemos hacer con los números 1, 2 y 3?

En el descanso de un partido de fútbol el marcador señalaba 0? 1. ¿De cuántas formas pudo ir


variando el marcador hasta llegar al resultado final de empate a 3 goles?

Con las letras de la palabra ALBA, ¿cuántas palabras, con o sin sentido, se pueden hacer?

Pablo tiene 5 pantalones y 15 camisas distintas, ¿de cuántas formas diferentes se puede vestir?

En cierto instituto se ofrecen 2 áreas optativas para 1º ESO, 3 para 2º ESO, 4 para 3º ESO y 5
para 4º ESO. ¿Entre cuántos itinerarios distintos puede elegir un alumno?

Si lanzamos 3 dados y una moneda, ¿cuántos resultados posibles podemos obtener?


Un restaurante dispone de 10 primeros platos, 8 segundos y 5 postres. ¿Cuántos menús
diferentes se pueden confeccionar?

Queremos crear un código que conste, en primer lugar, de una vocal, y a continuación de dos
cifras distintas elegidas entre el 1 y el 9? ambos incluidos? ¿Cuántos elementos distintos
podemos conseguir con este código?

¿Cuántas diagonales tiene un heptágono?

Juan tiene 20 € y decide participar en un juego que consiste en lanzar una moneda 4 veces. En
cada tirada debe apostar 20 €, que pierde si sale cruz. Si sale cara, gana 20 € más. Escribe
todos los resultados que pueden darse sabiendo que si se queda sin dinero concluye el juego.

Los 25 municipios de una ciudad están unidos a los demás por distintas líneas de tren. ¿Cuántas
líneas habrá en total?

A una fiesta acuden 6 parejas. Cada persona saluda con un abrazo al resto, menos a su
compañero/a. ¿Cuántos abrazos se han dado en total en la fiesta?

Dadas las letras A, B, C, E, indica cuántas ordenaciones se pueden hacer sabiendo que nunca
pueden ir juntas ni dos vocales ni dos consonantes.

PROBLEMAS DE CONTEO (PRIMARIA) NO INCLUYE FIGURAS

María tiene dos blusas y tres faldas. ¿De cuántas formas diferentes puede
combinarlas para vestirse?

En una escuela los alumnos tienen que elegir un deporte y un taller para cursarlos.
Los deportes que se ofrecen son: futbol, basquetbol, volibol y atletismo. Los talleres
son: carpintería, electricidad y mecanografía. ¿De cuántas formas distintas puede un
alumno combinar estas opciones? Acaba de llenar la siguiente tabla
Si a los deportes se agregara la natación y a los talleres la herrería, ¿cómo aumentaría
el número de combinaciones posibles?

Cinco amigos se encuentran en la calle y se saludan de mano. ¿Cuántos apretones


de mano hubo en total? ¿Y si hubieran sido 6, 7, 8, ... amigos?

Un torneo de tenis se realiza por eliminación simple, es decir, cada vez que se
enfrentan dos jugadores, el que pierde queda eliminado. Si en el torneo participan
64 tenistas, ¿cuántos juegos hacen falta para decidir quién es el campeón?

¿Cuántos partidos hay en un torneo de eliminación simple en el que participan n


equipos (para cada partido uno sale y el otro pasa a la siguiente etapa)?

Dos puntos determinan una recta; tres puntos, si no son colineales, determinan tres
rectas. Investiga lo que pasa con 4, 5, 6, ... puntos.

En un torneo de volibol participan 12 equipos de escuelas diferentes. Cada equipo


se enfrenta a otro dos veces, una vez como local y otra como visitante. ¿Cuántos
juegos se realizan en total?

7. Se quiere ir de una ciudad A a una ciudad M, pasando por las ciudades P y Q. De


A a P hay cuatro caminos, de P a Q hay dos y de Q a M hay tres. ¿De cuántas formas
diferentes se puede ir de A a M?
8. ¿Cuántas banderas de tres franjas y colores diferentes pueden hacerse si se dispone
de tela de cinco colores? ¿Y si se permite repetir un mismo color en franjas separadas?
9. De un grupo de cinco niñas y cuatro niños se va a escoger una niña y un niño para
formar una pareja de baile. ¿De cuántas formas diferentes puede integrarse la pareja?
10. ¿Cuántos resultados diferentes pueden obtenerse al lanzar dos dados cuyas caras
están marcadas con los números 1, 2, 3, 4, 5 y 6?
11. Respecto al problema 1 de la p. 67 podemos preguntarles cuántas formas de
vestirse tendría María si en lugar de dos blusas y tres faldas, tuviera tres blusas, tres
faldas y dos pantalones.
12. En el problema de los caminos, podemos decirles que actualmente se construyen
dos caminos más: uno de A a P y otro de Q a M y preguntarles cuántas formas de ir
de A a M habrá cuando se terminen, etcétera.
13. Elaboren una tabla donde aparezca el número de resultados diferentes que
pueden obtenerse al lanzar 1, 2, 3, ...., volados.
1. Encontrar todos los números de cuatro cifras que cumplan que las cifras de las
unidades y los millares sean iguales entre sí y que la suma de sus cifras sea 20.
2. ¿Cuántos números hay entre 0 y 100 que tengan al menos un 7 entre sus cifras? ¿Y
entre 0 y 1000?
3. ¿Cuál es el mayor número que puede formarse permutando las cifras de 745 (o de
3993)? ¿El menor? ¿Cuántos pueden formarse menores que 500 (o 5 000)? ¿Cuántos
pueden formarse en total?

El número de matrícula (placa) de un automóvil está formado por tres letras y


tres dígitos, incluido el cero. ¿Cuántas placas pueden hacerse con este sistema si las tres
letras pueden ir al principio o al final, pero no mezcladas con los dígitos? ¿Y si no se
permiten números con ceros al principio?

¿De cuántas maneras distintas se puede completar $1 utilizando moneda


fraccionaria? (Hay monedas de 5¢, 10¢, 20¢ y 50¢.)

En un restaurante, un parroquiano puede escoger entre dos sopas, cuatro guisados


y tres postres. ¿De cuántas formas diferentes puede componer su menú? Si se
quisiera aumentar el número de combinaciones posibles agregando un platillo, ¿qué
convendría aumentar: el número de sopas, el de guisados o el de postres?

Utilizando cada una de las cifras 1, 2, 3 y 4 se pueden escribir diferentes números, por ejemplo,
podemos escribir 3241. ¿Cuál es la diferencia entre el más grande y el más pequeño de los
números que se construyen así? (PI10)

(a) 2203 (b) 2889 (c) 3003 (d) 3087 (e) 3333

De la ciudad A a la ciudad B hay 3 caminos, de la ciudad A a la ciudad C hay 5 caminos, de la


ciudad B a la D hay 2 caminos y de la ciudad C a la D hay dos caminos. Si un camino que une
dos ciudades no pasa por otra, ¿cuántas formas hay de ir de la ciudad A a la D? (PI32)

(a) 12 (b) 16 (c) 19 (d) 32 (e) 60

Cinco amigos P, Q, R, S y T se dan la mano. Tanto P como Q estrecharon la mano de uno solo
de sus amigos, mientras que R, S y T estrecharon cada uno la mano de dos. Sabemos
que P estrechó la mano de T. ¿Quiénes podemos asegurar que no se dieron la mano? (PI57)

(a) T y S (b) T y R (c) Q y R (d) Q y T (e) Q y S

Omar le da a cada uno de sus libros una clave de tres letras utilizando el orden alfabético: AAA,
AAB, AAC,... AAZ, ABA, ABB, etc. Considerando el alfabeto de 26 letras y que Omar tiene
2203 libros, ¿cuál fue el último código que Omar utilizó en su colección? (PI86)

(a) CFS (b) CHT (c) DGS (d) DFT (e) DGU

¿Cuántos números enteros hay entre 9992 y 10002, sin incluir estos dos números? (PI102)

(a) 999 (b) 1000 (c) 1998 (d) 1999

Se tienen 9 ciudades y se quieren construir carreteras entre pares de ellas de tal forma que sea
posible viajar entre cualesquiera dos de ellas. ¿Cuál es el mínimo número de carreteras que se
deben construir? (PI130)

(a) 8 (b) 9 (c) 18 (d) 36


¿De cuántas maneras distintas pueden colorearse los lados de un triángulo equilátero con cuatro
colores distintos, si suponemos que un mismo color se puede emplear en lados distintos y que
dos coloraciones son iguales si difieren en un giro del triángulo en el plano? (PI144)

(a) 4 (b) 20 (c) 24 (d) 16


¿Cuántos números diferentes de cinco cifras se pueden formar con los dígitos 1, 1, 2, 2, 3?
(PI194)

(a) 120 (b) 40 (c) 30 (d) 20

En una clase hay 25 alumnos. Entre ellos 17 alumnos son ciclistas, 13 nadadores y 8
esquiadores. Ningún alumno hace tres deportes. Los ciclistas, nadadores y esquiadores se
sacaron 9 en matemáticas. Seis alumnos en la clase se sacaron 6 en matemáticas. ¿Cuántos
nadadores saben esquiar? (PI199)

(a) 2 (b) 4 (c) 6 (d) 10

Para numerar las páginas de un libro se necesitó utilizar 2004 dígitos. ¿Cuántas páginas tiene el
libro?

Luis Miguel compró una bolsa con 2000 caramelos de 5 colores; 387 eran blancos, 396
amarillos, 402 rojos, 407 verdes y 408 cafés. Decidió comerse los caramelos de la siguiente
forma: Sin mirar sacaba tres de la bolsa. Si los tres eran del mismo color, se los comía, si no, los
regresaba a la bolsa. Continuó así hasta que sólo quedaron dos caramelos en la bolsa. ¿De qué
color eran?

Una exposición se exhibe en 9 salas. Cada cuarto esta comunicado con sus cuartos
vecinos y el número indica cuantas veces paso José por cada uno de ellos. ¿Cuántas
veces paso José por el cuarto central? ROM13

¿Cuántos números enteros positivos pares menores que 2007 puedes formar con los
dígitos 0, 1, 2, 4, 6, 9, si se pueden repetir dígitos en un mismo número? ROM23

En un torneo de ajedrez cada jugador juega una vez con cada uno de los otros
participantes. No hay partidos nulos y ningún jugador le ganó a todos los otros
jugadores. ¿Hay algún jugador que pueda decir: “A cada jugador que me ganó, le ganó
otro jugador al cual yo le gané? ROM24

6. En un tablero de ajedrez se coloca una ficha en el cuadro de la esquina superior izquierda y se traslada hasta
la otra esquina opuesta. La ficha va de cuadro en cuadro (horizontalmente y verticalmente). No repetirá
cuadrados ya visitados. ¿Es posible que la ficha haya pasado por todos los cuadros del tablero? YUCATÁN
7. En una fiesta de monstruos, Frankestein saludó a sus amigos pero no saludó a sus enemigos. El Conde
Drácula saludó a todos los monstruos. La Momia se dio cuenta de que Drácula saludó al doble de monstruos
que Frankestein y también que saludó a 16 monstruos más que Frankestein. ¿Cuántos monstruos había en la
fiesta? YUCATÁN
8. Manuelita numeró las páginas de un libro de 100 hojas (cada hoja contiene 2 páginas), comenzando con el
#1. Vino Pedro y, furtivamente, arrancó 25 hojas. Luego sumó los 50 números escritos en ellos. ¿Es posible que
la suma de los números de las 25 hojas arrancadas sea 2004? Justifica tu respuesta. YUCATÁN

9. En un salón de clases de la escuela “Nachi Cocom”, 6 alumnos saben maya, 7 inglés y 5 francés. De éstos sólo uno
habla los tres idiomas. De los demás, 2 alumnos saben maya e inglés, 2 alumnos inglés y francés y 1 maya y francés.
Además todos ellos hablan español. ¿Cuántos alumnos hay en total en el salón que habla español? YUCATÁN

10. En un recinto cuadrado del zoológico hay nueve leones.

¿Cómo se puede conseguir, construyendo dos recintos cuadrados más, ubicar cada uno de los leones en un recinto
separado? YUCATÁN

12. Si el 29 de febrero de 1976 fue domingo, ¿En que año volverá a caer en domingo el 29 de febrero? YUCATÁN

13. ¿Cuántas veces aparece el número 5 entre los números del 1 al 1000? YUCATÁN

14. Construye el mayor número con las cifras 1,1,2,2,3,3,4,4 de tal forma que los dos “unos” estén separados por una
cifra, los “dos” por dos cifras, los “tres” por tres y los “cuatros” por cuatro cifras. Por ejemplo: los números (11223344),
(12341234),… YUCATÁN

21. Cada integrante de un grupo de 10 niños es amigo de exactamente 7 del grupo (la amistad es mutua). Pruebe que no
es posible dividirlos en 3 grupos de tal manera que en cada uno de los 3 equipos no haya un par de amigos. YUCATÁN

22. ¿De cuántas maneras distintas puedo escribir CHILAM BALAM siguiendo el esquema que se nos muestra (cuantos
caminos distintos hay para llegar de la C a la última M)? YUCATÁN

C
H H

I I I
L L L L

A A A A A

MMMMMM

B B B B B

A A A A

L L L

A A

¿De cuántas maneras se pueden colocar 4 fichas en un tablero de 5x5 de manera que
no haya 2 o más fichas en un mismo renglón o columna, y además, las 4 fichas estén
en casillas del mismo color? ROM40

La tienda de don Tomás es famosa por el color de sus canicas.


Don Tomás siempre las guarda en cajas de 100 canicas
del mismo color y tiene un bote pequeño en el que caben
exactamente 23 canicas. Un día llegó Luis y lo retó con el
siguiente problema.
Le pidió que llenara el bote con 23 canicas tomadas de su caja de
100 canicas blancas, que vaciara estas 23 canicas blancas en la caja
de 100 canicas rojas y las revolviera muy bien. Después de esto le
pidió que volviera a llenar su bote con 23 canicas del cajón bicolor
(con las canicas rojas y blancas revueltas) y que lo vaciara en el
cajón de las canicas blancas y revolviendo muy bien las canicas.
Luis le preguntó a don Tomás ¿qué hay más, canicas blancas en
el cajón de las canicas rojas, o canicas rojas en el cajón de las
blancas?
Justifi ca tu respuesta.

En un estado las placas tienen 3 letras y 3 números. El costo para hacer cada número
“n” es “$n” si n>0 y $10 para el dígito 0. Para las letras del costo es proporcional a la
posición de la letra en el alfabeto, es decir, $1 para la A, $2 para la B, y así hasta $26
para la Z. ¿Cuánto cuesta producir todas las placas empezando por la AAA000 hasta la
ZZZ999?

Un taxi puede llevar a 4 pasajeros: 1 enfrente y 3 atrás. ¿De cuántas formas se pueden
sentar 4 personas si una de ellas no puede ir adelante?

En un estado las placas tienen 3 letras y 3 números. El costo de hacer cada número n
es $n si n>0 y $10 para el dígito 0, Para las letras el costo es proporcional a la
posición e la letra en el alfabeto, es decir, $1 para la A, $2 para la B, y así hasta $26
para la Z. ¿Cuántas placas costarán exactamente 100 pesos?

¿Cuántos números enteros hay entre 1 y 1000 tal que la suma de sus dígitos es igual a
7?

Escribe todos los números mayores que 5000 que pueden obtenerse permutando
(intercambiando) las cifras del número 4507. Ordénalos de menor a mayor

30 RESOLUCIÓN DE PROBLEMAS DE CONTEO (SÍ INCLUYE FIGURAS)

1. Observa la siguiente imagen que representa un


mapa de carreteras que comunica los pueblos A,
B, C y D. Como puedes ver por la dirección de las
flechas, todas estas carreteras son de una sola
dirección.
a) ¿De cuántas maneras distintas puede realizar su
viaje del pueblo A al
D?_____________________________________
b) ¿Bastaría con sumar los caminos entre los pueblos? ¿Por
qué?_____________________________________________
c) ¿Cómo calcularías la cantidad de caminos diferentes que hay para ir del pueblo A al D sin
hacer un diagrama de árbol ni contando de uno en
uno?_______________________________________________
d) ¿Qué operación aritmética utilizarías para calcular el total de
rutas?_____________________________
e) Si se cierra uno de los cuatro caminos de B a C por mantenimiento, ¿cuántas rutas le
quedarían para ir de A a
D?________________________________________
_____________________________________
f) Si se cerrarán de manera alternada cada uno de
los caminos para darles mantenimiento, ¿cuál de
estos cierres afectaría más a los automovilistas al
reducir a la mitad el número de rutas para ir del
pueblo A al pueblo
D?________________________________________
_____

2. Ana vive en el centro de la ciudad de Puebla,


en la esquina que forman las calles 2 Norte y 6
Oriente. Ella va a la escuela que está ubicada
en 4 Norte y 12 Oriente. El mapa muestra el
recorrido que ayer hizo Ana para ir de su casa
a la escuela.
a) ¿Cuál es el menor número de cuadras que debe caminar Ana para ir de su casa a la
escuela?__________________
b) ¿Cuántas formas diferentes tiene Ana de caminar el menor número de
cuadras?_________________________
3. En “La gran rebanada” elaboran pasteles
de diferentes sabores, formas y
decorados. Cuando alguien hace un
pedido, el vendedor debe llenar un
formato como el siguiente:
a) ¿Cuántos pasteles diferentes pueden
elaborar en esa
pastelería?________________
b) ¿Habrá más de 10 pasteles
diferentes?____
c) ¿Más de
20?__________________________
d) ¿Más de
40?_________________________

4. Cuatro amigas compran números


consecutivos de boletos para ir al teatro,
¿de cuántas maneras diferentes pueden
sentarse?

Julia tiene en su ropero 3 faldas de colores blanco, rosado y marrón; 3 blusas de colores
negro, amarillo y azul, y 2 pares de zapatos de color marrón y negro. Combinando todas
estas prendas de vestir, ¿de cuántas maneras se puede vestir Julia? P3 OEJ 2012

¿Cuántos cuadritos de lados 1 1, 2 2, 3 3, ... podemos formar en una cuadrícula
de lados n n? Por ejemplo, en la siguiente cuadrícula de lados 8 8 están indicados
algunos cuadritos de lados 1 1, 2 2, 3 3,... LPM 258

3. ¿De cuántas maneras distintas pueden llenarse los cuadritos en blanco para que el
número resultante sea divisible entre 3 y entre 5?

4. Considera todos los números que pueden obtenerse permutando (cambiando


de lugar) las cifras de 8 025. ¿Cuántos son divisibles entre 2? ¿Entre 3? ¿Entre 5?
¿Entre 9?

Cinco ninos juegan a las escondidas en el patio de su escuela, cuatro se esconden y otro
los busca. En ese patio hay solo 3 escondites, los que diario usan: atras del arbol, atras
del bote de basura y bajo la banca (en donde caben dos ninos), Les toca esconderse a
Ana, Beto, Carlos y David. .De cuantas formas distintas se pueden repartir en los
escondites? P2 OEJ 2011

Si en una reunión se contaron en total 123 saludos de mano y cada uno de los asistentes saludó
exactamente a otros tres, ¿cuántas personas asistieron a la reunión? P10 OEJ 2012

¿Cuántas formas hay de llegar de A a B si no se puede pasar dos veces por el mismo punto?
(PI95)

(a) 10 (b) 12 (c) 16 (d) 18 (e) 20

El número de triángulos con sus tres vértices en los puntos de la figura es: (PI75)

(a) 20 (b) 24 (c) 28 (d) 32 (e) 36

Con vértices en los puntos de la figura, ¿Cuántos cuadriláteros se pueden dibujar? (PI109)

(a) 4 (b) 16 (c) 24 (d) 36

En el siguiente cubo, ¿de cuántas formas se puede ir de A a B sobre las aristas sin pasar dos
veces por el mismo vértice y sin subir? (PI118)
(a) 10 (b) 11 (c) 12 (d) 13

¿Cuántos triángulos hay en la figura?

(a) 22 (b) 20 (c) 18 (d) 14

En la siguiente figura cada vértice puede tomar el valor 1 ó -1, ¿cuántos valores distintos puede
tomar la suma A + B + C + D + E + F + ABCDEF? (PI45)
(a) 14 (b) 8 (c) 7 (d) 4

En la siguiente figura, cuántas rutas distintas puede seguir una hormiga para ir de la esquina A a
la esquina B, si sólo puede caminar sobre las líneas negras, y sólo se puede mover hacia arriba y
hacia la derecha.

¿Cuántos cuadrados hay en una cuadrícula de 3 x 3? ¿Y en una de 4 x 4?

Se quiere pintar cada región de la figura que se muestra con un solo color: de Rojo (R) o de Azul
(A). Se puede colocar la letra R o la letra A en cada región de tal modo que la figura puede ser
coloreada de diferentes maneras. ¿De cuántas formas diferentes se puede colorear la figura?
P13 OEJ 2013
La siguiente figura es un tablero de ajedrez. Calcule el número total de cuadros que hay
(de 1X1, 2X2, 3X3, 4X4, 5X5, 6X6, 7X7,
8X8). ROM16 // P21 OEJ 2011

En una mesa hay 5 monedas como se muestra en la figura. ¿En qué orden fueron
colocadas? ROM30

Con las siguientes placas se ha escrito “con todas sus letras” el número 1 310:

a) Encuentra todos los números que pueden escribirse combinando de diferentes


formas las cuatro placas anteriores. Ordénalos de menor a mayor.
b) Si se dispone además de otra placa con la palabra siete , ¿cuáles son todos los
números que pueden escribirse utilizando las cinco placas?
PRIMER TALLER DE RESOLUCIÓN DE PROBLEMAS OLÍMPICOS

Se escriben los dígitos 1, 2, 3, 4 en cuatro papelitos que se guardan en una caja. Si dos de los
papelitos se extraen al azar, ¿cuál es la probabilidad de que la suma de los números sea múltiplo
de 3?

Numeré 2014 tarjetas del 1 al 2014 y quité aquéllas que terminaban con 3. Después volví a
numerar las que me quedaban y por último quité las que terminaban en 2. Al final, ¿cuántas
tarjetas me quedaron?

¿Cuántos números menores que 100 se pueden escribir usando los dígitos 2, 3 y 5?

Con las cifras 1, 3, 4, 5 y 6, ¿cuántos números de cuatro cifras distintas se podrán formar de
modo que acaben en cifra par?

A una familia de 6 personas les ha tocado un viaje para dos personas. ¿De cuántas formas se
pueden repartir el viaje?

¿Cuántos ángulos de 30° están dibujados en un hexágono regular con todas sus diagonales
trazadas?

Carlos tiene una colección de 18 palillos. La colección tiene tres palillos de 1 cm, tres de 2 cm,
tres de 3 cm, tres de 4 cm, tres de 5 cm y tres de 6 cm. ¿Cuántos triángulos distintos puede
formar Carlos utilizando tres palillos de su colección?
¿Cuántos números de tres dígitos existen tales que la suma de esos tres números dígitos sea
24?

Con 0, 1, 2, 3 y 4, ¿cuántos números de cinco cifras se pueden formar, sin repetir ningún dígito?

Cuando se escriben los números 1,2,3,4,5,6,7,8,9,10,11,12,13,14,15,16,17,... ¿cuál es el dígito


que ocupa la posición 2002? Nota: en la lista anterior el dígito siete (de 17) ocupa la posición
25.

¿Cuántas intersecciones formarán diez rectas en el plano sino hay tres de ellas concurrentes?

PROBABILIDAD

Clase 1: “Carrera de autos” con dos dados.

1. Al lanzar dos dados, ¿cuáles son todos los posibles resultados?


a. Complete la tabla:
1 2 3 4 5 6

6
2. Organízate con once compañeros más para jugar dos veces el juego.

Los equipos deben ser de mínimo 8 y máximo 12 jugadores.

Materiales: Tablero (proporcionado por el profesor), dos dados de puntos de distinto color (con
puntos que representen los números del uno al seis).

• Preparen el tablero del Anexo, dos dados de diferente color, y 12 fichas o piedritas que
sirvan para marcar el avance de cada jugador.
• Cada jugador toma una ficha y la coloca en la casilla del auto con el que desea competir. Si
dos o más participantes seleccionan el mismo auto, pueden decidir quién escoge primero
mediante un volado. A cada jugador le corresponde un carro diferente.
• Por turnos, cada integrante del equipo irá lanzando los dados y el auto que tenga el mismo
número que la suma de los puntos del tiro, avanza una casilla rumbo a la meta.
• Gana el auto que llegue primero a la meta.

Es importante advertir que los resultados (1,4) y (4,1) son diferentes, pues se trata de dados
diferentes, por ello es necesario utilizar dados de diferente color, o en su defecto, de distinto
tamaño

Preguntas de discusión:

1. ¿Qué autos ganaron en las dos rondas?____________________________________


2. Si jugaran una tercera ronda, ¿qué auto convendría seleccionar?__________________
¿Por qué?_____________________________________________________________

Todos los posibles resultados se pueden apreciar en la siguiente tabla.

SUMA 1 2 3 4 5 6 7 8 9 10 11 12

S
FRECUENCIA

Clase 2: “Carrera de autos” con dominó

Organízate con once compañeros más para


jugar dos veces el juego.

Los equipos deben ser de mínimo 8 y máximo 12


jugadores.

Materiales: Tablero (proporcionado por el profesor), dos dados de puntos de distinto color (con
puntos que representen los números del uno al seis).

Se cambian los dados por fichas de dominó. Básicamente el juego se realizaría siguiendo las
mismas reglas, con el mismo número de integrantes por equipo y utilizando el mismo tablero.
Pero, para este juego los alumnos tendrían que considerar que por las características de las
fichas, las oportunidades para cada número son diferentes, inclusive, el auto con el número uno
sí tendría posibilidad de avanzar.

Para jugar esta versión, se entrega a cada equipo un juego de dominó sin la ficha llamada
“blanca” o “mula güera” (0,0), así, la suma de los puntos varía de uno a 12. Las fichas se
colocan con los puntos hacia abajo. Por turnos, cada integrante toma una ficha y suma los
puntos; el resultado es el número del auto que avanza una casilla.

Es importante que las fichas que vayan saliendo se regresen al montón para revolverlas. De la
misma forma que en la primera versión, cuando un auto llega a la meta el juego termina.

Los dos factores que modifican los resultados son:

Ahora los números que se pueden sumar son siete (0, 1, 2, 3, 4, 5, 6).
A diferencia de los dados, en las fichas solamente hay una forma de combinar dos números.
Por ejemplo (1,2) y (2,1) representan dos resultados diferentes al lanzar dos dados, en
cambio (1,2) y (2,1) representan un solo resultado en el dominó, es decir, es la misma ficha.

1. ¿Qué autos ganaron en las dos rondas?_______________________________________


2. Si jugaran una tercera ronda, ¿qué auto convendría seleccionar?___________________
¿Por qué?_________________________________________________________________
______________________________________________________________________

Todos los posibles resultados se pueden apreciar en la siguiente tabla.

SUMA 1 2 3 4 5 6 7 8 9 10 11 12

FRECUENCIA

EVENTOS ALEATORIOS: ¿MÁS O MENOS PROBABLE?

1. Se tienen 11 casillas numeradas del 2 al 12. Se lanzan dos dados y la suma de ambos indica el
número que avanza una casilla. Gana la carrera quien llegue primero a la meta.

10 META

3
2

2 3 4 5 6 7 8 9 10 11 12

SALIDA

a. ¿Todos los números tienen la misma probabilidad de ganar? No, porque unas
combinaciones se repiten más que otras.
b. ¿Qué número tiene más probabilidades de ganar? ¿Cuál menos probabilidades? El 7
tiene más probabilidades de salir y el 2 y el 12, tienen menos probabilidades.
c. Si pudieran elegir, ¿qué número o números no escogerían? R: El 2 ó el 12, porque
tienen menos probabilidades de salir.

2. Obtén el espacio muestral del juego anterior. Demuestra las respuestas que diste en el problema
anterior. Consideren que, por ejemplo (1,6), no es lo mismo que (6,1).

Suma 2 3 4 5 6 7 8 9 10 11 12

Espacio (1,1) (1,2) (1,3) (1,4) (1,5) (1,6) (2,6) (3,6) (4,6) (5,6) (6,6)
muestral
(2,1) (2,2) (2,3) (2,4) (2,5) (3,5) (4,5) (5,5) (6,5)

(3,1) (3,2) (3,3) (3,4) (4,4) (5,4) (6,4)

(4,1) (4,2) (4,3) (5,3) (6,3)

(5,1) (5,2) (6,2)

(6,1)

Frecuencia 1 2 3 4 5 6 5 4 3 2 1

Probabilida 1 de 2 de 3 de 4 de 5 de 6 de 5 de 4 de 3 de 2 de 1 de
d de salir la 26 36 36 36 36 36 36 36 36 36 36
suma

a. ¿Qué suma de números menores que 6, es más probable que caiga? ¿Cuál es la menos
probable? Más probable el 5 y menos probable el 2
b. Al lanzar dos dados, ¿qué suma de números mayor que 9 es más probable que caiga?, ¿cuál
es la menos probable? Más probable el 10 y menos probable el 12.

3. Al lanzar un dado, ¿qué es más probable?


a. ¿Que caiga número par o impar?
b. ¿Que caiga número primo o compuesto?
c. ¿Que caiga múltiplo de 2 ó de 3?
d. ¿Que caiga múltiplo de 3 ó de 4?
REPASO: 08 EVENTOS ALEATORIOS: ¿MÁS O MENOS PROBABLE?

1. Al lanzar dos dados, ¿cuál de los siguientes eventos tiene mayor probabilidad de ocurrir?
A) “Que la suma de los números que salgan sea par”.

B) “Que se obtenga 2 o 3 en alguno de los dados”.

C) “Que la suma de los números que salgan sea menor o igual que 7”.

D) “Que el producto de los números que salgan sea par”

2. Seguramente conoces el juego llamado “serpientes y escaleras”. En éste, cada jugador avanza tantas
casillas como los puntos que caen en la cara superior de un dado que previamente lanzó. Si cae en la
base de una escalera, sube a la casilla correspondiente; si cae en la cola de una serpiente, baja hasta
la casilla donde llega la cabeza. Gana el que llegue primero a la casilla 100. Juan y Ana están jugando,
Juan está en la casilla 95 y le toca lanzar el dado, ¿cuáles son los posibles números que deben caer
para que gane el juego en esta tirada?
A) (1,4)(4,1)(3,2)(1,3) B) (1,4)(4,2)(3,2)(2,3)

C) (1,4)(4,1)(3,1)(2,3) D) (1,4)(4,1)(3,2)(2,3)

3. Juan y Pedro acuerdan una apuesta en el lanzamiento de dos monedas. Juan gana si caen águilas y
sellos. Pedro gana en caso de que caigan sólo águilas o sellos. ¿Quién de los dos tiene mayor
probabilidad de ganar y porqué?
A) Juan, porque son dos monedas y caerán dos águilas y dos sellos siempre.

B) Pedro, porque deben caer águilas y sellos, en ese orden.

C) Juan, porque pueden caer más águilas y sellos.

D) Los dos, porque los dos lanzamientos tienen la misma oportunidad de salir.

4. Una bolsa contiene 50 canicas del mismo tamaño: 10 blancas, 15 rojas, 20 azules y 5 negras. Si cierro
los ojos y saco una, ¿de qué color es más probable que sea?
A) Azul. B) Blanca. C) Negra. D) Roja.

5. Una TV se va a rifar con 100 boletos numerados del 1 al 100. ¿En cuál de las siguientes opciones es
más probable que se encuentre el número ganador?
A) En los números pares. B) En los múltiplos de cinco.
C) En los múltiplos de tres. D) En los diez números iníciales.

6. Juan lanza un dado dos veces. ¿Cuál de los siguientes resultados es más probable que caiga?
A) Que caigan números pares. B) Que caigan números entre 1 y 4.

C) Que caigan números menores que 3 D) Que caigan números mayores o iguales a 2.

7. Al lanzar al mismo tiempo un dado y una moneda al aire, ¿cuál es la probabilidad de que caiga un
número par y un sol?
A) 1/4 B) 1/3 C) 1/12 D) 3/4

8. Al lanzar dos dados, ¿qué es más probable?


A) Qué salgan números iguales en los dados B) Que salgan números diferentes en los dados

C) Que la suma sea par D) Que la suma sea menor a 4

9. Para obtener puntos en un juego de azar, es necesario lanzar dos dados y que la suma de sus caras
sea un número par. ¿Cuál de las siguientes afirmaciones es correcta? Justifica tu respuesta en tu
cuaderno.
A) Cualquier número tiene la misma probabilidad de salir.

B) Es más probable que en ambos dados caiga número impar.

C) Es más probable que en ambos dados caiga número par.

D) El número con menor probabilidad de salir es el 2.

10. En un salón utilizaron tres urnas con bolitas de colores para realizar una experiencia aleatoria. La urna
1 tiene 6 rojas, 6 verdes y 3 amarillas. La urna 2 tiene 4 rojas, 6 verdes y 2 amarillas. La urna 3 tiene
3 rojas, 5 verdes y 1 amarilla. Selecciona la opción que es correcta.
A) Es menos probable que se extraiga una bola roja de la urna 1, que de la 3.

B) Es menos probable que se extraiga una bola amarilla de la urna 2, que de la 1.

C) Es menos probable que se extraiga una bola verde de la urna 3, que de la 1.

D) Es menos probable que se extraiga una bola roja de la urna 2, que de la 3.

REACTIVOS ETAPA DE ZONA

Marcos tiene todas las letras del


abecedario en tres tamaños: grandes,
medianas y pequeñas:
Usando letras de dos tamaños, Marcos quiere escribir el nombre de su amiga ANA. ¿De cuántas
maneras puede hacerlo?

¿Cuántas parejas de enteros positivos impares tienen como suma 20014?

Un número capicúa es aquel número que se lee igual de derecha a izquierda que de izquierda a
derecha, por ejemplo 12321, 5775, ¿Cuántos números capicúas de 3 dígitos existen que sean
par?

Erick escribe todos los números entre 100 y 2014 que cumplan con estas dos
condiciones:
a) La cifra de las centenas es igual a la cifra de las unidades
b) La suma de sus dígitos es un número par.

¿Cuántos caminos hay del punto A al punto B siguiendo las líneas de la forma si las direcciones

permitidas son es decir, cualquier sentido está permitido

excepto y no se permite pasar dos veces por el mismo punto?

¿Cuántos números de tres cifras hay tales que la suma de las primeras dos cifras es igual a la
tercera cifra?
¿Cuántos números de cuatro cifras y menores que 2014 cumplen estas condiciones: son pares,
son múltiplos de 3 y no son múltiplo de 5? Explica cómo los contaste.

Se tienen dos dados. En las caras de uno de ellos aparecen los números 2, 4, 8, 16, 32 y 64,
mientras que en las caras del otro aparecen los números del 1 al 6. Tiramos los dados y
multiplicamos los dos números que obtengamos. ¿Cuál es la probabilidad de que esta
multiplicación sea un cuadrado perfecto?

Luis y Elena van a formar cada uno de ellos un número de tres cifras. Para ello, eligen
alternativamente un dígito cada uno entre los números 1, 2, 3, 4, 5 y 6 (los dígitos no se pueden
repetir).
Luis gana si el número formado es múltiplo de 3. En caso contrario, gana Elena.
Si eligen los números al azar, ¿qué probabilidad de ganar tendría Luis?

¿Cuál es el digito que ocupa el lugar 2014 en el desar rollo decimal de 4/101?

Queremos crear un código que conste, en primer lugar, de una vocal, y a continuación de dos
cifras distintas elegidas entre el 1 y el 9, ambos incluidos. ¿Cuántos elementos distintos
podemos conseguir con este código?

En un aula hay 100 alumnos, de los cuales: 40 son hombres, 30 usan gafas, y 15
son varones y usan gafas; si seleccionamos al azar un alumno de dicho curso.
a) ¿Cuál es la
probabilidad de que sea mujer y no use gafas?
b) Si sabemos que el alumno seleccionado no usa gafas, ¿qué probabilidad hay de que sea
hombre?

Mi bicicleta esta asegurada por una cadena con un candado de código, el número que abre el
candado esta formado por tres cifras tal que su producto es impar y la suma de estos dígitos es
cuadrado perfecto. ¿Cuántos códigos existen y cuales son?

¿Cuántos números de dos dígitos (es decir, números mayores que 9 y menor
es que 100)
cumplen que la suma de sus dígitos es par?

¿Cuántos productos de tres cifras iguales o distintas podemos hacer con los números 1,2 y 3?

TAMAULIPAS

Víctor escribió en una lista todos los números que pueden formarse revolviendo
los dígitos 2, 0, 1, 3 (sin repetir ninguno). Los números quedaron escritos de mayor
a menor. Después se calcularon las diferencias entre cada dos números
consecutivos de la lista, siempre restando a cada número el que le sigue en la lista.
¿Cuál es la mayor de estas diferencias?
En una reunión hay 201 personas de 5 nacionalidades diferentes. Se sabe que, en cada grupo
de 6, al menos dos tienen la misma edad. Demostrar que hay al menos 5 personas del mismo
país, de la misma edad y del mismo sexo.

Dados tres cubos unidos como ilustra la figura, halla el número de caminos desde
A hasta B, considerando que cada camino se compone de exactamente 5 aristas.

Si en una reunión se contaron en total 123 saludos de mano y cada uno de los
asistentes saludó exactamente a otros tres, ¿cuántas personas asistieron a la
reunión?

Julia tiene en su ropero 3 faldas de colores blanco, rosado y marrón; 3 blusas de


colores negro, amarillo y azul, y 2 pares de zapatos de color marrón y negro.
Combinando todas estas prendas de vestir, ¿de cuántas maneras se puede vestir
Julia?
En una mesa hay 5 cartas. Cada carta tiene de un lado un número natural y del otro una letra. Juan afirma: Cualquier
carta que tenga de un lado una vocal tiene un número par del otro lado. Pedro demostró que Juan mentía dando vuelta
sólo a una carta. ¿De cuál de las cinco cartas se trata?

Un costal está lleno de canicas de 20 colores distintos. Al azar se van sacando canicas del costal.
¿Cuál es el mínimo número de canicas que deben sacarse para poder garantizar que en la
colección tomada habrá al menos 100 canicas del mismo color? (en el costal hay más de 100
canicas de cada uno de los 20 colores) (PI2) // P6 OEJ 2011

(a) 1960 (b) 1977 (c) 1981 (d) 1995 (e) 2001

Luis Miguel compró una bolsa con 2000 caramelos de 5 colores; 387 de eran blancos, 396
amarillos, 402 rojos, 407 verdes y 408 cafés. Decidió comerse los caramelos de la siguiente
forma: Sin mirar sacaba tres de la bolsa. Si los tres eran del mismo color, se los comía, si no, los
regresaba a la bolsa. Continuó así hasta que sólo quedaron dos caramelos en la bolsa. ¿De qué
color eran? (PI78)

(a) Blancos (b) Amarillos (c) Rojos (d) Verdes (e) Cafés

Una bolsa contiene 8 fichas negras y las demás son rojas. Si la probabilidad de sacar una
ficha roja es de 2/3, ¿cuántas fichas hay en la bolsa?

¿Cuál es el mínimo número de estudiantes que debe de haber en un salón para asegurar que
hay al menos 10 niños o al menos 10 niñas?

FUENTES DE INFORMACIÓN

OMM RECURSOS

http://ichi.fismat.umich.mx/omm/recursos/

PI: PROBLEMAS INTRODUCTORIOS

http://ichi.fismat.umich.mx/omm/recursos/prob15/index.html
ECA: EXAMEN CANGURO ANIMADO

http://ichi.fismat.umich.mx/omm/recursos/canguro/anima/

ROM: Reactivos para Olimpiadas de Matemáticas

http://www.redexperimental.gob.mx/olimpiadasfis_mat/ProblemasMatematicas.pdf

LPM: Libro para el maestro Matemáticas. Educación Secundaria. SEP.

http://www.imc-official.org/en_US/history/kimc2014/

www.librosmaravillosos.com/matematicarecreativa/capitulo10.html

1. Problemas de Combinatoria. Entrenamiento OMM Guanajuato 2003

2. Cuadernillos de entrenamiento. Olimpiada Estatal de Matemáticas en Jalisco. 2010 a 2013

3. Primer Taller de Resolución de Problemas Olímpicos (Martha Catalina Guzmán Reyes)

4. Reactivos para la Fase de Zona de la 2ª Olimpiada Estatal Sinaloa 2014 (Martha Guzmán)

5. Problemas Introductorios (César Octavio Pérez Carrizales).

6. Repasín de combinatoria (César Octavio Pérez Carrizales).

7. http://www.acertijosyenigmas.com/

8. http://acertijos.elhuevodechocolate.com/

9. http://www.matmor.unam.mx/clubmate/

10.http://www.oma.org.ar/programa/index.htm (Olimpiada Matemática Argentina)

11.http://educa-creando.blogspot.mx/2011/05/libros-de-olimpiadas.html

12.http://revistasuma.es/IMG/pdf/58/SUMA_58.pdf

13.http://www.masquemates.com/canguro.html (Canguro ESPAÑA)

También podría gustarte